You are on page 1of 120

1.

A 75-year-old woman who has had type-2 diabetes mellitus for the last 15
years is admitted for cataract surgery. She is taking metformin 500 mg bid,
plain insulin 10 U at night, ramipril 2.5 mg od and bendroflumethiazide
(bendrofluazide) 2.5 mg od. She drinks a glass of whisky to help her sleep at
night. Investigations showed: BM 3.0 mmol/l; urea 10 mmol/l; creatinine 140
mol/l; Na+ 130 mmol/l; K+ 3.7 mmol/l.
Liver function tests (LFTs) were normal and arterial blood gas measurements
showed a pH 7.3, p(CO2) 5.1 kPa and HCO3 17 mmol/l.

Which one of the following is the most detrimental in these circumstances and should
be stopped?

Ramipril

Metformin

Your answer

Insulin

Alcohol

Bendroflumethiazide (bendrofluazide)

Metformin is a biguanide used as a blood glucose lowering agent. The major toxicity of
metformin is lactic acidosis. It should not be used in patients with renal insufficiency (creatinine
> 133 mol/l in males and > 124 mol/l in females), any form of acidosis, congestive heart
failure, liver disease or severe hypoxia. As it is metabolised in the liver, it is contraindicated in
patients with liver disease or a high alcohol intake. Ramipril, an angiotensin-converting enzyme
inhibitor (ACEI), is indicated for the treatment of diabetic nephropathy with albuminuria. It may
potentiate the hypoglycaemic effect of insulin and oral antidiabetic agents, especially during the
first few weeks of use. It should be used under specialist supervision if the creatinine
concentration is > 150 mol/l, and renal function and potassium levels monitored. This patient
has long-standing diabetes with nephropathy. Hypoglycaemia is common with nephropathy and
is thought to be due to the decreased metabolism of insulin. Decreasing renal function can
cause metabolic acidosis, either of which is a contraindication for treatment with metformin.
Bendroflumethiazide (bendrofluazide) can be safely continued.
2.

A 35-year-old man with type-1 diabetes mellitus on insulin presents in the


A&E with fever, cough, vomiting and abdominal pain. Examination reveals a
dry mucosa, decreased skin turgor and a temperature of 37.8 C. Chest
examination reveals bronchial breathing in the right lower lobe, and a chest Xray shows it to be due to a right lower zone consolidation. Other investigations
show a blood glucose concentration of 30 mmol/l, Na+ 130 mmol/l, K+ 5.7
mmol/l, urea 8.0 mmol/l, creatinine 120 mol/l, pH 7.15, HCO3 12 mmol/l,
p(CO2) 4.6 kPa and chloride 106 mmol/l. Urinary ketones are positive (+++).
The patient is admitted to the hospital and treated.
Which of the following should not be used while treating him?

Fluids, iv

Insulin

Potassium

Your answer

Bicarbonate

Correct answer

Antibiotics

This patient has diabetic ketoacidosis (DKA), which is more common in type-I than type-2
diabetes mellitus. Inadequate insulin causes lipolysis, leading to ketosis and acidosis. The
common precipitants include inadequate insulin administration, infections (pneumonia, urinary
tract infection (UTI)), myocardial infarct (MI), stroke, peripheral gangrene, etc. Treatment is
directed at the underlying cause and correction of the ketoacidotic state. Here, the underlying
cause, pneumonia, is treated with antibiotics. The most important aspect of treatment is to give
iv fluids, as normal saline, along with regular insulin. When the fluid balance is restored, the
initial high potassium concentration may fall and therefore has to be monitored and
supplemented as required.
The role of HCO3 in DKA is controversial. The acidosis usually corrects itself once the fluid and
electrolyte balance is restored. Some physicians administer HCO 3 if severe acidosis (pH < 7) is
present. However, there is a view that rapid correction of acidosis may impair cardiac function.
3.

A 30-year-old woman is evaluated in the endocrinology clinic for increased


urine output. She weighs 60 kg and has a 24-hour urine output of 3500 ml.
Her basal urine osmolality is 210 mOsm/kg. She undergoes a fluid deprivation
test and her urine osmolality after fluid deprivation (loss of weight 3 kg) is
350 mOsm/kg. A subsequent injection of subcutaneous DDAVP (desmopressin
acetate) did not result in a further significant rise of urine osmolality after 2
hours (355 mOsm/kg).
Which of the following is the likely diagnosis?

Normal

Primary polydipsia

Correct answer

Osmotic diuresis

Pituitary diabetes insipidus

Nephrogenic diabetes insipidus

Your answer

Diabetes insipidus (DI) is suspected when the urine output is > 50 ml/kg per day (3000 ml for a
60-kg female). If the basal urine osmolality is > 300 mOsm/kg, it suggests an osmotic diuresis.

If fluid deprivation causes a urine osmolality > 300 mOsm/kg, it suggests psychogenic (primary)
polydipsia. If not, it is either pituitary or nephrogenic DI. These are differentiated by the
administration of subcutaneous DDAVP which causes an increased urine osmolality > 300
mOsm/kg in pituitary DI.
4.

Which of the following is a feature of MEN-1?

Marfanoid features

Phaeochromocytoma

Medullary carcinoma of the thyroid

Your answer

Parathyroid hyperplasia

Correct answer

Mucosal neuromas

MEN-1 (multiple endocrine neoplasia type-1; Werners syndrome) involves the three Ps:
parathyroid, pituitary, pancreas.

Parathyroi
d

Hyperparathyroidism is the most common manifestation in MEN-1

Pituitary

Prolactinomas are most common in the pituitary gland

Pancreas

Second most common involvement in MEN-1. Pancreatic tumours are associated


with:

(a) Pancreatic polypeptide (7585%)

(b) gastrin (ZollingerEllison syndrome) recurrent peptic ulcers

(c) insulinoma hypoglycaemia

(d) glucagonoma hyperglycaemia and skin rash (necrolytic migratory erythema)

(e) VIPoma (vasoactive intestinal polypeptide-secreting tumour) Verner


Morrison syndrome or watery diarrhoea hypokalaemia achlorhydria (WDHA
syndrome)
MEN-2 usually involves the thyroid and parathyroid glands and phaeochromocytoma. The mode
of inheritance in MEN-1 is autosomal-dominant. MEN-2A is characterised by medullary thyroid
carcinoma, parathyroid hyperplasia/adenoma and phaeochromocytoma. In addition, MEN-2B
has mucosal and gastrointestinal (GI) neuromas and marfanoid features.
5.

A 35-year-old HIV-positive man, evaluated for weight loss and weakness has
been found to have disseminated tuberculosis. On examination, he is
hypotensive and has hyperpigmentation of the mucosa, elbows and skin
creases. Further investigations confirm a diagnosis of Addisons disease.

Which of the following is likely in this condition?

Increased serum Na

Increased serum Cl

Increased serum HCO3

Increased serum K

Your answer

Decreased serum Ca

In adrenal destruction, serum Na, Cl and HCO3 levels are reduced, and serum potassium is
elevated. The hyponatraemia is due to both loss of sodium in the urine (due to aldosterone
deficiency) and to movement into the intracellular compartment. Hyperkalaemia is due to a
combination of aldosterone deficiency, impaired glomerular filtration (due to hypotension) and
acidosis. Mild to moderate hypercalcaemia occurs in 1020% patients, the cause of which is
uncertain.
6.

A 60-year-old man is referred to the endocrine clinic with a complaint that his
shoe size has gone up from size 9 to size 11 and his wedding ring no longer
fits him. He is sweating a lot and his wife complains he is snoring more at
night.

Which of the tests below is most useful for confirming a diagnosis of acromegaly?

OGTT with GH measurements

Serum IGF-1 level

Your answer

Skull X-ray

Random GH level

MRI of the pituitary fossa

A 75-g oral glucose tolerance test (OGTT) is the gold standard for a diagnosis of acromegaly. In
normal people the growth hormone (GH) level suppresses during the test, but in patients with
acromegaly it is not suppressed. This is because insulin and GH are antagonistic hormones.
Random levels of GH and insulin-like growth factor-1 (IGF-1) may be raised, but as there is a
wide range of normal a single measurement is not sufficient to make the diagnosis. A dynamic
test (the OGTT) is required. Both a skull X-ray and an MRI scan are likely to be abnormal in
patients with acromegaly. Pituitary tumours in acromegaly are usually macroadenomas. As other
secretory and non-secretory tumours of the pituitary can also be macroadenomas these tests
would not confirm the patient had acromegaly, though they would be used in the diagnostic
work-up.
7.

A 62-year-old woman presents to her GP complaining of thirst and polyuria.


Her fasting glucose level is 9 mmol/l and 8.2 mmol/l on two separate
occasions, confirming she has diabetes mellitus.

Which of the following additional findings would be least likely to be associated with a
diagnosis of secondary diabetes mellitus?

A bitemporal visual field defect

A large goitre

Your answer

Hypertension

Multiple striae and bruises

Maculopathy

Correct answer

Secondary causes of diabetes occur in other endocrine conditions where hormones with
antagonistic actions to insulin are present in excess amounts. Patients with acromegaly, which
shows elevated growth hormone (GH) levels secondary to a pituitary adenoma, can also have a
goitre, visual field defects and hypertension and diabetes. Those with Cushings syndrome, from
glucocorticoid excess, can have visual field defects, hypertension, striae and bruising.
Maculopathy is present in diabetes of long duration and is related to long-term poor glycaemic
control. It could be present in patients with secondary diabetes if they have gone undiagnosed
for some time. However, maculopathy is not related to any of the hormone excesses seen in
these conditions.

8.

A 26-year-old woman attends her GP complaining of feeling tired all the time
for the last few months. She has had no period for 6 months and has been
feeling dizzy first thing in the morning.

Which of the following clinical signs would the GP be most likely to find if the
diagnosis was thought to be glucocorticoid deficiency?

Buccal pigmentation

Diminished body hair

Pallor

Postural hypotension

Your answer

Optic atrophy

Hypoadrenalism has many causes including Addison's disease, granulomatous disorders, TB,
tumour (particularly lung) or related to infection (meningococcal septicaemia). Secondary
hypoadrenalism is caused by hypopituitarism or hypothalamic disorders. Pallor is present in
hypopituitarism due to normochromic, normocytic anaemia and a lack of melanocyte-stimulating
hormone (MSH; cleaved from the precursor of ACTH). Buccal pigmentation is also associated
with glucocorticoid deficiency, most commonly from Addisons disease. ACTH is elevated in
Addisons disease and results in higher levels of MSH, which causes the skin and buccal
pigmentation found in this condition. Lack of body hair and amenorrhoea are features of
hypogonadism in hypopituitarism. Postural hypotension is related to glucocorticoid deficiency
regardless of cause.
9.

A 30-year-old woman presents to her GP with a history of amenorrhoea and


galactorrhoea. She is keen to become pregnant and has been trying for 9
months to conceive without success. She is of normal weight and has no other
constitutional symptoms.

Which of the following is most likely to be the reason for her symptoms?

Drug treatment she is on for asthma

Hypothyroidism

Hyperthyroidism

Previously undiagnosed hepatic impairment

Pituitary microadenoma

Your answer

Her symptoms are consistent with hyperprolactinaemia. Raised prolactin has many causes
including pituitary tumours (micro- and macroadenomas), drug treatment (some
antidepressants like tricyclic antidepressants, anti-emetics and cimetidine), pregnancy, stress,
hypothyroidism (because thyrotrophin-releasing hormone (TRH) stimulates TSH and prolactin in
primary hypothyroidism) and renal failure (prolactin accumulates in renal failure but not in
hepatic impairment). Hyperthyroidism is not associated with hyperprolactinaemia and
galactorrhoea, although it can cause amenorrhoea and fertility problems. While
hyperprolactinaemia can be present in hypothyroidism it is usually associated with menorrhagia
rather than amenorrhoea.
10. An 18-year-old young man presents to his GP with thirst and polyuria. Some 6
months previously he had a significant head injury as the result of a road
traffic accident. He is referred to the local endocrine clinic.
Which of the following results would be the most useful in confirming a diagnosis of
diabetes insipidus after a water deprivation test?

Plasma sodium of 126 mmol/l

Plasma sodium of 150 mmol/l

Plasma osmolality of 335 mOsm/kg and urine osmolality of 700


mOsm/kg

Plasma osmolality of 280 mOsm/kg and urine osmolality of 700


mOsm/kg

Your answer

Plasma osmolality of 335 mOsm/kg and urine osmolality of 200


mOsm/kg

Correct answer

Diabetes insipidus (DI) can be due to cranial DI with complete or partial deficiency of
antidiuretic hormone (ADH, vasopressin), or renal DI due to renal tubular defects affecting the
action of ADH on water reabsorption from the renal tubules. The plasma sodium can be normal
or elevated, depending on whether the patient can continue to drink freely and so prevent
dehydration. A low plasma sodium concentration is associated with SIADH (syndrome of
inappropriate ADH). The diagnostic test to confirm DI is a water deprivation test. The inability to
concentrate the urine during the test results in the plasma osmolality rises and the urine
osmolality remaining dilute. Normal plasma osmolality is 285305 mOsm/kg. The higher the
urine osmolality the more concentrated it is.
11. A 56-year-old lifelong smoker presents to his GP with a history of cough,
breathlessness and weight loss. A chest X-ray is abnormal with a mass at the
right hilum. Which of the following results is most likely to suggest the tumour
is a small-cell lung tumour?

Serum calcium of 3.3 mmol/l

Serum sodium of 123 mmol/l

Your answer

Serum potassium of 5.5 mmol/l

Plasma osmolality of 335 mOsm/kg

Urine osmolality of 145 mOsm/kg

Small-cell lung tumours can secrete a number of hormones including ADH (vasopressin) and
ACTH. Excess ADH causes SIADH (syndrome of inappropriate ADH) with hyponatraemia and low
plasma osmolality (normal 285305 mOsm/kg) and concentrated urine (> 500 mOsm/kg).
Excess ACTH would cause a hypokalaemic alkalosis and not hyperkalaemia. Hypercalcaemia
occurs in squamous-cell lung tumours because of ectopic parathyroid hormone (PTH) secretion.
12. A 54-year old woman is seen for the first time in the diabetes clinic. She is
obese, plethoric and has marked bruising on her limbs and fresh striae over
her abdomen. She has a dorsal kyphosis following a vertebral collapse earlier
in the year.
Which of the following results will help to pinpoint the diagnosis if you suspect
Cushings syndrome secondary to adrenal adenoma?

Normal 0900-h serum cortisol level

Serum potassium of 2.2 mmol/l

0900-h serum cortisol of 200 nmol/l after overnight dexamethasone test

Raised urine cortisol/creatine ratio

Undetectable serum ACTH level

Your answer

Cushings syndrome can be ACTH-dependent (pituitary-driven Cushings, ectopic ACTH


secretion) or ACTH-independent (adrenal adenoma, adrenal carcinoma and adrenal nodular
hyperplasia). The serum cortisol level can be normal in all these conditions as it has a wide
range of normal. There is a loss of diurnal variation and night-time cortisol is elevated. Serum
potassium is most likely to be low in cases of ectopic ACTH or adrenal carcinoma. The urine
cortisol/creatinine ratio is elevated in all causes of Cushings and doesnt help to differentiate the
cause. A normal ACTH level can be found in pituitary-driven Cushings and sometime also in
ectopic ACTH as there is overlap between the normal and elevated ranges. In adrenal causes of

Cushings syndrome the ACTH is suppressed or undetectable. All causes of Cushings syndrome
would give an unsuppressed 0900-h cortisol level after an overnight dexamethasone
suppression test.
13. A 67-year-old man is admitted with a 6-week history of proximal muscle
weakness. He has been having difficulty climbing stairs and getting up from a
sitting position in a chair. He is a lifelong smoker and alcohol intake is 30 units
per week.
Which of the following results is most likely to point to a diagnosis?

Abnormal liver function tests and macrocytosis

Correct
answer

A suppressed 0900-h serum cortisol level after an overnight


dexamethasone suppression test

Your answer

Hypercalcaemia

Low serum vitamin B12 level

Elevated TSH and normal free-thyroxine levels

The symptoms he is describing are in keeping with a proximal myopathy. Causes include alcohol
excess (abnormal LFTs and raised MCV), Cushings syndrome (failure to suppress 0900-h serum
cortisol level after an overnight dexamethasone suppression test), osteomalacia (low calcium
and raised alkaline phosphatase levels) and thyrotoxicosis (suppressed TSH and raised free T 4
levels) or hypothydrodism (elevated TSH and low FT4). The TFTs above are in keeping with
compensated hypothydrodism which isn't associated with proximal myopathy. Low vitamin B 12
(eg in pernicious anaemia) causes a number of neurological symptoms including peripheral
neuropathy and subacute combined degeneration of the spinal cord, but not proximal myopathy.
14. A 51-year-old man presents to A&E with altered consciousness, his blood
pressure is 80/50 mmHg, his skin is pigmented and he has a past history of
Hashimotos thyroiditis. His family says he has been tired for several months
and has been losing weight and complaining of abdominal pain.
Which of the following results is most likely to be found on investigation?

Laboratory glucose level of 12.3 mmol/l

Serum sodium level of 116 mmol/l

Peaked T-waves on ECG

Your answer

Serum potassium level of 2.4 mmol/l

Serum urea level of 3.2 mmol/l

The clinical picture is suggestive of Addisons disease with hypotension, pigmentation and a
prodromal period with symptoms in keeping with glucocorticoid deficiency in a man with a
history of another autoimmune endocrine disorder. Other disorders that can be associated are
diabetes and pernicious anaemia. Hypoglycaemia, hyponatraemia, hyperkalaemia and an
elevated urea are all present during a hypoadrenal crisis due to steroid deficiency and
subsequent salt and water loss. Although the potassium is elevated it is unusual to find peaked
T-waves on the ECG in patients with hypoadrenalism: the complexes tend to be small, lowvolume ones.
15. A 31-year-old man is referred to the local hypertension clinic because of
recently discovered hypertension that is labile and difficult to control.
Which of the following features is most likely to suggest a genetic/familial syndrome
is the cause for his hypertension?

Serum potassium level of 3.9 mmol/l

Your answer

Random blood glucose level of 9 mmol/l

A serum calcium level of 1.5 mmol/l

A family history of unexplained death in childbirth

Correct answer

A family history of papillary thyroid neoplasia

Adrenal causes of hypertension are Conns syndrome (primary hyperaldosteronism), Cushings


syndrome (from adrenal adenoma, carcinoma or adrenal nodular hyperplasia) and
phaeochromocytoma. Hypokalaemia is a feature of Conns and Cushings syndromes.
Hyperglycaemia can be present in Cushings syndrome or phaeochromocytoma. Unexplained
death in childbirth and a family history of thyroid neoplasia raise the possibility of multiple
endocrine neoplasia type-2 (phaeochromocytoma, medullary thyroid tumours and
hyperparathyroidism) as the cause of his hypertension. A raised serum calcium level would also
point towards MEN2 (multiple endocrine neoplasia type 2) but hypocalcaemia is not associated
with adrenal causes of hypertension.
16. A 25-year-old woman presents to a reproductive endocrinology clinic with a
history of being unable to conceive after 2 years of using no contraception. It
is thought she may have polycystic ovarian syndrome.
Which of the following is most likely to be associated with this condition?

A 28 day menstrual cycle

Elevated LH/FSH ratio

Your answer

Normal free-androgen index

Low levels of circulating insulin

Normal BMI (body mass index)

Polycystic ovarian syndrome is one of the commonest causes of anovulatory infertility. Patients
can have a normal menstrual cycle but are more likely to have oligomenorrhoea. It is associated
with a number of biochemical abnormalities, including raised LH levels, normal or elevated
testosterone but with a low SHBG (sex-hormone-binding globulin) resulting in a high freeandrogen index. Androstenedione levels can either be normal or raised. The underlying
biochemical defect in patients with PCOS is recognised to be insulin resistance. This causes high
circulating insulin levels due to peripheral insulin resistance: therefore hyperinsulinaemia, and
not low insulin levels, is characteristic of the condition. The insulin resistance has been shown in
both lean and obese patients with the condition.
17. A 37-year-old woman presents to the endocrine clinic with a history of
hirsutism, acne and oligomenorrhoea. She is having difficulty losing weight
and has searched the Internet and thinks she may have polycystic ovarian
syndrome. She wants to discuss the implications of this.
Which of the following is the most important issue to discuss with her at this stage of
her life?

Exercise regimens

Does she want to have children

Correct answer

Her blood glucose level

Treatment for her hirsutism

Your answer

Weight-reduction diets

All the above are relevant and each should be discussed. The hirsutism and acne can be very
difficult to deal with and can cause distress from a cosmetic point of view. There are a variety of
treatment options but Dianette (cyproterone acetate) is probably the most effective, along with

cosmetic treatments like waxing, shaving, plucking or electrolysis. Her future risk of type-2
diabetes and associated cardiovascular risk is very important and she should be advised about
the need for lifestyle treatments and the need to lose weight and exercise regularly to reduce
the chance of this happening. It is now recommended that all patients with PCOS have their
fasting blood glucose level measured annually to pick up diabetes at an earlier stage. However,
the most important issue in a woman of her age is fertility, as women with PCOS frequently
require assistance with conception. Her age is against her if she is going to have problems with
fertility and requires help to conceive. The commonest treatment is to induce ovulation with
clomifene. She needs to be advised that pregnancy also carries an increased risk of gestational
diabetes.
18. A 29-year-old woman presents to her GP with a history of weight loss, heat
intolerance, poor concentration and palpitations.
Which of the following is most likely to be associated with a diagnosis of
thyroiditis associated with viral infection?

Bilateral exophthalmos

Diffuse, smooth goitre

Reduced uptake on thyroid isotope scan

Your answer

Negative thyroid peroxidase antibodies

Pretibial myxoedema

Graves disease is an autoimmune thyroid disorder and is more common in women than men. It
is associated with positive TPO (thyroid peroxidase antibodies) and TSH (thyroid stimulating
hormone) receptor antibodies. The goitre in Graves disease is normally diffusely enlarged rather
than nodular. Uptake of radio-isotope is increased in Graves disease. In other conditions causing
thyrotoxicosis, like postpartum thyroiditis and viral thyroiditis, the uptake is reduced. Graves
disease is associated with a number of other features not found in other causes of
thyrotoxicosis, eg exophthalmos, pretibial myxoedema and thyroid acropachy (a condition that
affects the nails and looks like finger clubbing). The exophthalmos can be unilateral or bilateral.
19. A 34-year-old woman is referred to the endocrine clinic with a history of
thyrotoxicosis. At her first appointment she is found to have a smooth goitre,
lid lag and bilateral exophthalmos with puffy eyelids and conjunctival
injection. She wants to discuss treatment of her thyroid problem as she is
keen to become pregnant.
What is the most likely treatment you would advise?

1218 months of carbimazole alone

1218 months of propylthiouracil alone

Correct answer

A combination of antithyroid drugs and thyroxine

Radioactive iodine (iodine-131)

Thyroidectomy

Your answer

All the above treatments are recognised treatments for thyrotoxicosis. Antithyroid drugs
(carbimazole and propylthiouracil) can be used alone and the dose titrated down or up to keep
the patient euthyroid. Treatment is usually for 1218 months. However carbimazole has been
associated with aplasia cutis (a scalp defect) in babies born to mothers treated during
pregnancy. The block and replace regimen of thyroxine and anti-thyroid drugs is not used in
pregnancy as thyroxine does not cross the placenta. Surgery is an option for people with a large
goitre, or for people who fail on other treatments but would not be first choice in a young
otherwise healthy woman. Radio-iodine is a very effective treatment for thyrotoxicosis but is
contraindicated in women of childbearing age who are contemplating pregnancy within 6
months.
20. A 30-year-old man and his wife present to a reproductive endocrinology clinic
because of infertility. The man is tall with bilateral gynaecomastia.
Examination of the testes reveals bilateral, small, firm testes.
Which of the following investigations is most helpful in diagnosing a patient with
Klinefelters syndrome?

CT scan of the pituitary gland

Chromosomal analysis

Your answer

Measurement of serum gonadotrophins

Measurement of serum testosterone

Semen analysis

Klinefelters syndrome is a genetic disorder with an extra X chromosome, giving a genotype XXY.
It is usually diagnosed in late pubertal or early adult life because of delayed sexual development
or infertility. It is associated with hypogonadism giving raised gonadotrophin levels and low
testosterone levels. Semen analysis would show azoospermia. Gynaecomastia is often present.
CT brain scan will be normal. The other tests are appropriate for investigation of male infertility.
There are other cause of infertility that would give high gonadotrophin and low testosterone
levels; any primary testicular disorder would give this pattern, eg cryptorchidism, mumps
orchitis, haemochromatosis, myotonic dystrophy and alcohol abuse. The only test that is specific
for Klinefelters syndrome is a genetic test for the XXY genotype.

21. A 26-year-old man is referred for gastroscopy because of several months of


dyspepsia. He has routine bloods checked and is found to have a serum
calcium level of 3.2 mmol/l with a venous bicarbonate level of 33 mmol/l.
Renal and liver function are both normal. Chest X-ray is normal.
What is the most likely cause of his hypercalcaemia?

Myeloma

Metastatic malignancy

Milkalkali syndrome

Your answer

Primary hyperparathyroidism

Sarcoidosis

All the above diagnoses are causes of hypercalcaemia. Symptoms of hypercalcaemia are
commonly thirst, polyuria, dyspepsia, malaise, bone pain and constipation. Myeloma and
metastatic malignancy cause hypercalcaemia either by direct lytic lesions of bone or by the
production of PTHrP (parathyroid hormone-related protein). Myeloma would be extremely rare in
this age group. There is nothing in the history to suggest bone pain or a diagnosis of metastatic
malignancy. Primary hyperparathyroidism is most commonly found by chance in elderly women.
It can be part of the multiple endocrine neoplasia syndromes and therefore can be seen in
younger people. The normal chest X-ray suggests that sarcoidosis is not the diagnosis in this
case. The age of the patient, the dyspeptic symptoms and the raised bicarbonate level suggest
the most likely diagnosis is the milkalkali syndrome caused by the ingestion of antacids.
22. A 17-year-old young woman is referred to the endocrine clinic with primary
amenorrhoea. She is of normal height and weight. She has moderate
hirsutism. A male cousin was seen in the clinic at the age of 8 years with
precocious puberty.
What is the most likely cause for her primary amenorrhoea?

Congenital adrenal hyperplasia

Hyperprolactinaemia

Polycystic ovarian syndrome

Turners syndrome

Testicular feminisation syndrome

Your answer

Turners syndrome (genotype XO) is associated with primary amenorrhoea and short stature.
Hyperprolactinaemia and PCOS are more commonly associated with secondary amenorrhoea.
Testicular feminisation causes primary amenorrhoea and is caused by either a partial or
complete androgen-receptor defect. Affected patients are genotypically male (XY) but
phenotypically female. Congenital adrenal hyperplasia is most commonly caused by 21hydroxylase deficiency. Severe forms present in infancy with salt-losing crises and females can
have ambiguous genitalia. Milder forms cause precocious puberty in boys and virilism, hirsutism
and primary amenorrhoea in girls. Diagnosis is by finding raised serum 17-hydroxyprogesterone
levels that show a hyperresponsiveness to ACTH. Treatment is with glucocorticoids.
23. An 81-year-old woman is referred to the thyroid clinic with increasing size of
a pre-existing goitre. She has had long-standing hypothyroidism and has been
on a dose of thyroxine of 100 g daily for many years.
Which of the following primary thyroid cancers is she most likely to have?

Anaplastic thyroid cancer

Follicular thyroid cancer

Medullary thyroid cancer

Papillary thyroid cancer

Your answer

Thyroid lymphoma

Correct answer

Primary thyroid lymphoma is strongly associated with lymphocytic thyroiditis (Hashimotos


disease), which is present in 80% of cases. It is predominantly a disease of elderly women. Up
to 30% of patients have a history of goitre and may be taking thyroxine. Anaplastic thyroid
cancer is commonly found in elderly patients and usually presents with a thyroid mass. It can
occur in a pre-existing goitre. It is usually rapidly progressive and has the worst prognosis of
the thyroid cancers. Medullary thyroid cancer arises from within the C cells of the thyroid, cells
which produce calcitonin. It is commonly associated with multiple endocrine neoplasia type-2
(MEN2) along with primary hyperparathyroidism and phaeochromocytoma. Follicular thyroid
cancer is typically found in middle-aged to older individuals and usually presents as a solitary
asymptomatic thyroid nodule. Eventually 20% of patients will develop distant metastases.
Papillary thyroid cancer is most commonly found in the 3050 age group but can occur at any
age. It most commonly metastasises to the local lymph nodes within the neck. Prognosis is
better than the other thyroid cancers.
24. A 52-year-old woman sees her GP complaining of a 4-kg weight gain, dry hair
and skin, she feels slow and always has the heating up high. She has a small
diffuse goitre. Her TSH level is 20 mU/l and free thyroxine 5 pmol/l. Thyroid
peroxidase antibody is positive at high titres. She has two sisters who have
had thyroid disease.
What is the most likely diagnosis?

DeQuervains thyroiditis

Follicular carcinoma

Hashimotos thyroiditis

Your answer

Graves disease

Nodular goitre

Hashimotos thyroiditis is an autoimmune thyroid disorder found more often in women than
men. It is associated with positive thyroid antibodies and there may be a goitre that tends to be
diffuse rather than multinodular. The gland is infiltrated with lymphocytes and patients can
become hypothyroid. DeQuervains thyroiditis is not associated with positive antibodies and
tends to present with painful swelling in the neck. Graves disease is also an autoimmune
thyroid disorder but patients usually present with thyrotoxicosis. Follicular carcinoma can
present with a thyroid swelling but not hypothyroidism.
25. A 45-year-old man presents to his GP with headaches. His BP is 166/94
mmHg. Routine investigations reveal sodium 142 mmol/l, potassium 2.6
mmol/l, chloride 101 mmol/l and normal urea and creatinine levels. Plasma
renin is undetectable and aldosterone levels are raised. What is the most likely
cause for his hypertension?

Cushings syndrome

Conn's syndrome

Your answer

Phaeochromocytoma

Renal artery stenosis

Acromegaly

Conns syndrome is a condition where there is a benign adrenal adenoma that is secreting
aldosterone. The renin level is low, and hypokalaemia is one of the common findings as well as
hypertension. Cushings syndrome, acromegaly and phaeochromocytoma are all associated with
secondary hypertension, but not with low renin and elevated aldosterone levels. Renal artery
stenosis has high renin levels as well as high aldosterone levels and so causes secondary
hyperaldosteronism. Treatment of Conns syndrome is with surgical excision of the adenoma or
with potassium-sparing diuretics.

26. You are called to see a 36-year-old woman on the surgical ward who is 2 days
post-thyroidectomy. She is complaining of tingling around her mouth and in
her hands and has developed spasm of her hands.
What immediate treatment can you give that is most likely to resolve her symptoms?

Ask her to breathe into a paper bag

Intravenous calcium

Your answer

Intravenous diazepam

Intravenous glucose

Intravenous potassium

Postoperative complications of thyroidectomy include recurrent laryngeal damage, haemorrhage


and inadvertent removal of the parathyroid glands, although actually less than 1% of patients
suffer permanent hypocalcaemia. The symptoms described are typical of hypocalcaemia,
although they can be seen in patients who are hyperventilating. The treatment of choice to
relieve the acute symptoms is 10 ml of 10% calcium gluconate. Hypocalcaemia can be transient
after thyroid surgery. If it is permanent then the patient needs long-term therapy with
alfacalcidol to maintain her calcium levels.
27. A 28-year-old woman who is 3 months postpartum comes to the surgery
complaining of tiredness, she has had no periods since the baby was born and
she has been unable to breast-feed because of a lack of milk production. You
notice in her case sheet that she required a blood transfusion after delivery for
postpartum haemorrhage.
What is the most likely diagnosis?

Empty sella syndrome

Nelsons syndrome

Prolactinoma

Sheehans syndrome

Sipples syndrome

Your answer

Sheehans syndrome is hypopituitarism following pregnancy complicated by haemorrhage at the


time of delivery. The haemorrhage and associated hypotension is thought to cause pituitary
infarction. Patients fail to lactate and remain amenorrhoeic postdelivery, and also develop other
pituitary hormone deficiencies of the thyroid and adrenal axis. Nelsons syndrome is seen in
patients with pituitary-driven Cushings syndrome many years after bilateral adrenalectomy.
Such patients are deeply pigmented. Empty sella syndrome is associated with hypopituitarism,
imaging of the pituitary fossa shows no obvious pituitary tissue. The cause is unknown. Sipples
syndrome is MEN2 (hyperparathyroidism, medullary thyroid tumours and phaeochromocytoma).
28. A 34-year-old man presents to the diabetes clinic with a history of thirst,
polyuria and a recent 3.2 kg (7 lb) weight loss. His urine contains a small
amount of ketones.
Which of the following would suggest he is most likely to have type-2 rather
than type-1 diabetes?

A BMI of 23

High circulating insulin level

Your answer

HLA type DR-3

Positive islet-cell antibodies

Plasma bicarbonate level of 8 mmol/l

Type-1 diabetes mellitus is an autoimmune disorder characterised by the presence, in many


patients, of autoantibodies to the islet cell (ICA), insulin (IAA) or glutamic acid dehydrogenase
(GAD). It is associated with HLA DR-3 or DR-4. Patients with type-2 diabetes more commonly
have a high BMI > 25, as this form of diabetes is associated with insulin resistance and high
insulin levels rather than low insulin levels. Finally, patients with type-2 diabetes mellitus can
have + or ++ ketones in the urine, but dont usually have severe acidosis represented by a
bicarbonate level of 8 mmol/l. This is more common in type-1 diabetes, with diabetic
ketoacidosis still a common and sometimes fatal complication.
29. A 67-year-old man is referred to the hospital diabetes clinic with a new
diagnosis of type-2 diabetes mellitus. He has a BMI of 29. Creatinine level is
150 nmol/l and he has 1+ protein on urinalysis. He has a past history of heart
failure.
Which of the following drugs are you most likely to prescribe?

Chlorpropamide

Gliclazide

Pioglitazone

Your answer

Rosiglitazone

Metformin

Gliclazide is a sulphonylurea and can be used alone or in combination therapy for type-2
diabetes mellitus. It acts by increasing insulin release from the pancreatic -cell. It can be used
in mild to moderate renal failure. Pioglitazone and rosiglitazone are thiazolidinediones, which
promotes insulin sensitivity by their action on the PPAR receptor (peroxisome proliferator
activated receptor-).They are associated with fluid retention and are contraindicated in heart
failure. Chlorpropamide, a sulphonylurea, is rarely used now and is excreted by the kidney.
Metformin is a biguanide and, although its mechanism of action is not entirely clear, it reduces
insulin resistance and hepatic glucose production. It is thought to be able to cause lactic acidosis
in certain circumstances and its use is contraindicated in patients with renal, hepatic or cardiac
failure.
30. A 27-year-old woman with type-1 diabetes mellitus attends for her routine
review and says she is keen on becoming pregnant.
Which of the following is the factor most likely to make you ask her to defer
pregnancy at this stage?

Minor background retinopathy

Hb A1C 9.4%

Your answer

She hasnt been taking folic acid

Microalbumin level of 6-mg excretion in 24 hours

Sensory neuropathy

Pregnancy in type-1 diabetes is still associated with a two- to threefold increase in congenital
abnormalities when compared with the background population. It is also associated with higher
neonatal morbidity and mortality and higher operative delivery rates. Prepregnancy counselling
with the aim of bringing the Hb A1c to near-normal levels is associated with better pregnancy
outcomes. Diabetic retinopathy can progress during pregnancy and the eyes must be checked at
least once each trimester; if there is progression that is sight-threatening then the patient
should be referred to an ophthalmologist. Background retinopathy would not be a
contraindication to pregnancy. The microalbumin level is within normal limits. Nephropathy can
also progress during pregnancy, but microalbuminuria would not be a contraindication to
pregnancy and nor would sensory neuropathy. It is important that patients with diabetes take 5
mg folic acid daily prepregnancy (this reduces neural tube defects) and this can be started when
the patient starts trying to conceive, although it should ideally be taken for 3 months

beforehand. The most important thing this woman can do is to improve her glycaemic control
before trying to conceive.
31. A 17-year-old young woman with poorly controlled diabetes mellitus presents
with a temperature, dehydration and altered consciousness. Her initial
biochemistry shows sodium 130 mmol/l, potassium 4.5 mmol/l, bicarbonate 6
mmol/l, urea 11.2 mmol/l, creatinine 135 nmol/l and hydrogen ion 80.
What is the most important immediate treatment?

Intravenous antibiotics

Intravenous bicarbonate

Intravenous fluids

Your answer

Intravenous insulin

Intravenous potassium

The patient has diabetic ketoacidosis, a condition still associated with mortality in patients with
type-1 diabetes mellitus. There is a total body deficit of fluid and electrolytes. The most
important initial treatment is intravenous saline as the hypovolaemia is the factor most likely to
cause the patient to die. Intravenous insulin is required at an early stage; and, although it is
normal, the potassium level will rapidly fall with fluid and insulin treatment and the patient will
require potassium replacement. Intravenous bicarbonate should not be used immediately as it
can cause massive fluid shifts and precipitate cerebral oedema. It can be used if hydrogen ion is
greater than 100 and the patient is not responding to initial measures, in which case small
volumes of a 1.26% bicarbonate solution should be used.
32. A 54-year-old man, newly diagnosed with type-2 diabetes mellitus, presents to
the clinic for his first assessment. He is found to have changes in his eyes on
fundoscopy.
Which of the following is most likely to need immediate referral to the
ophthalmologist?

A few dot and blot haemorrhages

Some hard exudates > 1 disc diameter from the fovea

Cataract

New vessels on the disc

Your answer

Two soft exudates in the temporal field

Background diabetic retinopathy consists of dot and blot haemorrhages and hard exudates.
Patients do not need to be referred to the ophthalmologist unless these are within 1 disc
diameter of the fovea. This can be monitored annually at the routine clinic. Cataracts appear
about 10 years earlier in type-2 diabetes than in non-diabetic patients. If the vision is
significantly affected the patients warrant routine and not urgent referral to the ophthalmologist.
Soft exudates suggest retinal ischaemia, which would require routine referral to the
ophthalmologist. New vessels anywhere in the fundus are a feature of proliferative retinopathy
and, as new vessels have a risk of haemorrhage and can threaten sight, they should be referred
urgently to the ophthalmologist.
33. A 78-year-old man is admitted to hospital with a left hemiparesis and altered
consciousness. He is on aspirin 75 mg, bendrofluazide 2.5 mg, atorvastatin 10
mg and glibenclamide 15 mg daily. His wife says he has been unwell for a
couple of days and has been off his food. She has still been giving him all his
medication.
Which of the following tests is going to be most helpful in finding an immediately
reversible cause for his symptoms?

Blood glucose level

Your answer

CT brain scan

ECG

Serum creatinine level

Troponin level

Hypoglycaemia in the elderly is a not uncommon problem, even if only treated with oral
hypoglycaemic agents. Glibenclamide has a long half-life and should therefore be avoided in the
elderly. The reduced food intake and the ongoing intake of medication in this patient is likely to
have caused hypoglycaemia, which can be associated with neurological symptoms in the elderly.
The neurological symptoms will resolve promptly with intravenous dextrose. Although a
myocardial infarction or a cerebrovascular accident could have caused his symptoms, they would
not be immediately reversible.
34. A 43-year-old woman presents with weight loss, palpitations, diarrhoea and a
cessation of periods. She has been treated by her GP for anxiety. Examination
reveals a single nodule on the left of her thyroid, about 1.5 cm in diameter.
Thyroid scanning with technetium shows increased uptake within the nodule
with reduced activity throughout the rest of the gland. Thyroid function tests
showed a free thyroxine of 30 pmol/l (925 pmol/l), TSH < 0.05 mU/l (0.5
5).
Based upon these findings, what would be the definitive treatment?

Radioactive iodine therapy

Correct answer

Carbimazole

Surgical excision

Your answer

Propanolol therapy

High-dose carbimazole therapy with thyroxine replacement

Toxic thyroid nodules preferentially take up radioactive iodine. This makes them particularly
amenable to radioactive iodine treatment. A dose of 300500 MBq is usually sufficient to cure
the thyrotoxicosis. Radioiodine therapy is contraindicated in children and women who are
lactating or pregnant, and where the safety of cohabitants from the effects of radioactivity cant
be guaranteed. Surgery is the next best option for patients in whom radioiodine is
contraindicated or refused. Carbimazole alone or in combination with thyroxine replacement is
used in the medical treatment of Graves disease; Propanolol is a useful symptom reliever for
thyrotoxicosis.
35. A 37-year-old woman presents to A&E after chasing thieves who were stealing
her car. Her blood pressure is noted to be 185/110 mmHg on admission. She
admits to episodic headaches and feeling stressed and anxious. She is of
normal appearance, her serum calcium on admission is noted to be 2.95
mmol/l with normal renal function, abdominal ultrasound reveals a possible
adrenal mass.
What is the most likely diagnosis?

Phaeochromocytoma

Your answer

Carcinoid syndrome

MEN-1

MEN-2a

Correct answer

MEN-2b

MEN-2 (multiple endocrine neoplasia type 2) is associated with medullary thyroid carcinoma
(almost always), parathyroid chief-cell hyperplasia (1025%) and phaeochromocytoma (20

50%). MEN-2b is also associated with a marfanoid appearance, whereas MEN-2a is not. The
cause is an autosomal-dominant inherited genetic mutation on the long arm of chromosome 10.
Correct management of this patient includes appropriate a- and -blockade prior to surgical
removal of the phaeochromocytoma, screening for medullary thyroid carcinoma with the iv
pentagastrin test and calcitonin assay, then likely thyroidectomy, and probable
parathyroidectomy in the hands of an experienced surgeon. The prognosis of MEN-2 is variable,
but, overall, the 10-year survival rate is around 65%.
36. A 24-year-old adopted man presents with transient left-sided weakness of his
arm, which resolves after a few hours. His only other history of note is a
reduced libido and inability to maintain erections. On examination he appears
to have a spotty skin pigmentation. You notice a heart murmur, and there is
suggestion of a left atrial mass on echo. His prolactin is elevated at 2000
mol/l.
What is the most likely diagnosis?

Left atrial myxoma

Carney complex

Your answer

Prolactinoma

Protein C deficiency

Somatisation disorder

This man has spotty skin pigmentation, probable prolactinoma and a probable left atrial
myxoma. Carney complex is diagnosable with two features out of spotty skin pigmentation,
myxoma, endocrine tumours (commonest being primary pigmented nodular adrenocortical
disease), but it is also associated with Sertoli-cell tumours, growth hormone- or prolactinproducing pituitary adenomas, thyroid adenomas and ovarian cysts) and psammomatous
melanotic schwannoma (PMS). It is also diagnosable on the presence of one feature and an
affected first-degree relative. It is an autosomal-dominant condition caused by an inactivating
mutation of protein kinase A on chromosome 17.
37. A 28-year-old man presents to casualty with a sudden loss of vision in his
right eye. His only past history of note is a previous cerebellar haemorrhage.
On examination he has evidence of bilateral retinal angiomas and a partial
retinal detachment in his right eye.
What is the most likely diagnosis?

Simple traumatic retinal detachment

Clotting disorder

Bleeding due to hypertension

von HippelLindau disease

Your answer

McCuneAlbright syndrome

Von HippelLindau disease is characterised by CNS and retinal haemangioblastomas (presenting


feature in 40% of patients), renal cysts and carcinomas (occurring later), phaeochromocytoma
(20% of affected families, bilateral in 40% of patients) and pancreatic tumours (50% nonfunctioning). The prevalence is 1 in 39,000 with a mean age at presentation of 27 years. Renal
cysts occur later, with 70% of patients having them by age 60 years. Ideally, genetic testing in
affected families should take place around the age of 5 years. Affected individuals then require
yearly urinalysis, catecholamine screening, and fluorescein angiography, with 3-yearly brain MRI
scanning.
38. A 32-year-old woman presents with collapse. She works in an office
environment and it has been a particularly hot day. On examination in casualty
she looks a little dehydrated. BP is 110/70 mmHg.
The following serum electrolyte results are obtained;
Na+ 134mmol/l
K+ 2.9mmol/l
Mg++ 0.57 mmol/l (0.75-1.05)
HCO3- 34mmol/l
What is the most likely diagnosis?

Bartter's syndrome

Gitelman's syndrome

Correct answer

Gordon's syndrome

Conn's syndrome

Liddle's syndrome

Your answer

Gitelmans syndrome is due to a mutation in the thiazide-sensitive NaCl transporter in the


distal convoluted tubule. It is associated with hypokalaemia, hypomagnesaemia (a cardinal
feature) and raised serum bicarbonate levels. There is also hypocalciuria. Treatment is with
potassium and magnesium replacement with or without potassium-sparing diuretics.
Differentiation between Gitelman's and Bartter's syndromes is made by measuring the urinary
excretion of magnesium (which is high in Gitelman syndrome and normal in Bartter syndrome)
and calcium (which is high in Bartter syndrome and low in Gitelman syndrome). Bartters
syndrome presents earlier with hypokalaemic alkalosis and hypercalciuria, due to a mutation in

the bumetanide-sensitive Na+K+-2Cl- transporter. Treatment is with potassium replacement,


non-steroidal anti-inflammatories and sometimes ACE inhibitors. Liddles syndrome is due to a
mutation in the distal nephron sodium channel, and is associated with hypokalaemic alkalosis
and low renin and aldosterone levels, but hypertension still occurs. Essentially, Gordons
syndrome is the opposite of Bartters syndrome, presenting with hyperkalaemia. Conns
syndrome is associated with hypertension and is due to aldosterone excess.
39. A 45-year-old woman who works in a pharmacy presents with episodes of
tiredness and lethargy. Her blood pressure is 115/75 mmHg. Her bloods reveal
hypokalaemia and a raised serum bicarbonate level. Urine collection reveals
hypercalciuria. Otherwise the findings are unremarkable.
What is the likely diagnosis?

Bartters syndrome

Gitelmans syndrome

Your answer

Frusemide abuse

Correct answer

Conns syndrome

Liddles syndrome

This picture could fit Bartters syndrome, although Bartters syndrome is rare (1 per million), has
an autosomal-recessive pattern of inheritance and commonly presents below the age of 5 years.
Features of Bartters syndrome include volume depletion, seizures, tetany and muscle
weakness. This womans occupation is the clue, which unfortunately makes frusemide abuse
very much more likely.
40. A 42-year-old man is referred to the hypertension clinic for advice. He is
currently taking atenolol, bendrofluazide and ramipril and his blood pressure
is currently 165/105 mmHg. His potassium is 3.0 mmol/l, with a serum
bicarbonate concentration of 28 mmol/l.
What is the best next management step?

Measure the aldosterone:renin ratio

Wash out as many of his antihypertensive agents as is possible for a


period of 2 weeks, then review

Measure his 24-h blood pressure

Your answer

Correct
answer

Arrange 24-h urinary free-cortisol collection

Add in a further agent and review in 12 months

The suspicion here with hypokalaemia and metabolic alkalosis, and resistant hypertension on
three agents, is that he has primary hyperaldosteronism. ACE inhibitors and angiotensin IIreceptor blockers, diuretics, calcium-channel blockers, b -blockers all ideally require a washout
period of 2 weeks to make the aldosterone: renin ratio assay meaningful. Whilst it may seem
dangerous to wash out, a definitive diagnosis is impossible if you don't. Spironolactone requires
a washout period of 6 weeks. A high aldosterone: renin ratio is suggestive of primary
hyperaldosteronism. The blood sample should be taken in the morning, standing, and with a
normalised potassium concentration (using supplementation) if possible. Urinary potassium
excretion > 30 mmol/24 h may be another useful adjunct in making the diagnosis. Surgery is
the treatment of choice for Conns adenoma and leads to resolution of hypertension in around
70% of patients, mitotane may be useful for controlling symptoms of adrenal carcinoma,
spironolactone is the medical treatment of choice for adrenal hyperplasia.
41. A 42-year-old man is referred to the hypertension clinic for advice. He is
currently taking atenolol 100 mg, bendrofluazide 2.5 mg and ramipril 10 mg,
and his blood pressure is currently 165/105 mmHg. Examination is otherwise
unremarkable. His potassium is 3.0 mmol/l, with a serum bicarbonate
concentration of 28 mmol/l, creatinine 85 mol /l, glucose tolerance is
normal.
What is the most likely underlying diagnosis?

Cushings disease

Primary hyperaldosteronism

Your answer

Essential hypertension

Renal artery stenosis

Phaeochromocytoma

There is evidence of hypokalaemic metabolic alkalosis, probably associated with primary


hyperaldosteronism. His resistant hypertension in the presence of marked hypokalaemia
supports this. Primary hyperaldosteronism accounts for at least 2% and perhaps up to 10% of
hypertensive patients. The aldosterone: renin ratio is likely to be raised if the patient is off
antihypertensive medication, as is urinary potassium excretion. It is important, however, to do
the blood testing in the morning, after a period of standing and wash out of this mans
antihypertensives for a period of 2 weeks if possible. Hyperaldosteronism does respond to some
extent to spironolactone, but surgery is the treatment of choice for Conns adenoma. Bilateral
adrenal hyperplasia, glucocorticoid-suppressible hyperaldosteronism and adrenal carcinoma are
other causes.

42. A 24-year-old student has been recovering at home after a period of intensive
care and general medical admission for meningococcal septicaemia. 4 days
after discharge from hospital she visits her GP complaining of dizziness on
standing and profound tiredness. On examination she looks tired and washed
out and does indeed have postural hypotension. Blood testing reveals a
sodium concentration of 121 mmol/l, potassium of 6.7 mmol/l and urea of
15.0 mmol/l. She has mild normochromic, normocytic anaemia.
What is the most likely underlying diagnosis?

Secondary adrenal insufficiency

WaterhouseFriderichsen syndrome

Your answer

ME

Syndrome of inappropriate ADH

Hypothyroidism

WaterhouseFriderichsen syndrome is bilateral adrenal haemorrhage occurring due to massive


septicaemia, often associated with severe, life-threatening meningococcal disease. It may
present as in this patient with tiredness, lethargy and postural hypotension a short period after
discharge from the precipitating illness. Classically, laboratory findings associated with adrenal
insufficiency are hyponatraemia, hyperkalaemia, elevated urea, anaemia, elevated ESR,
eosinophilia and mild hypercalcaemia. The commonest cause in the western world is
autoimmune disease (70%), closely followed by malignancy, although TB is a common
association in the developing world. Emergency treatment of adrenal insufficiency involves fluid
replacement, IV hydrocortisone and glucose supplementation, often even prior to determining
the underlying cause.
43. A 38-year-old psychiatric patient who is being treated for depression is
admitted with increased confusion. His sodium is 122 mmol/l, with a plasma
osmolality of 235 mOsmol/kg. Urine osmolality is 300mOsm/kg. There is no
evidence on examination of cardiac, renal or hepatic failure. Random plasma
cortisol and thyroid function are normal.
Given the most likely diagnosis, which of the following statements best fits this
condition?

Reduced renal sodium excretion is likely

Treatment with IV saline is likely to be necessary

Renal sodium excretion is likely to be normal

Your answer

Psychiatric drugs are very unlikely to be related to the underlying


condition

Hypothyroidism is a possible contributor

This is the syndrome of inappropriate antidiuretic hormone (SIADH) secretion. Criteria for
diagnosis are hyponatraemia with an osmolality of less than 270 mOsmol/kg. Inappropriately
raised urine osmolality (> 100 mOsmol/l). High urinary sodium of > 20 mmol/l (reflecting
relative increased concentration due to reduced water loss), and normal renal, adrenal and
thyroid function. Sodium handling by the kidney is not affected in SIADH. Psychiatric drugs,
particularly monoamine oxidase inhibitors and phenothiazines may be possible causes. Other
possible causes include tumours, such as small-cell lung cancer, other chest pathology,
intracerebral haemorrhage or trauma, drugs such as carbamazepine or chlorpropamide and
metabolic causes such as hypothyroidism. Treatment is with fluid restriction or tetracycline-like
compounds that promote water excretion.
Another possible diagnostic consideration in a presentation of hyponatraemia, especially given
the psychiatric aspect here, is that of psychogenic polydipsia. However, in this condition urine is
maximally dilute (urine osmolality < 100mOsm/kg), in contrast to SIADH.
44. A 44-year-old patient with hypomania is referred for opinion. She is noted to
have a sodium concentration of 142 mmol/l, with a urea of 12 mmol/l and a
creatinine of 140 mol/l. Urine osmolality is 250 mOsmol/l. Fasting plasma
glucose is normal. The nurses have monitored her urine output and found it to
be 4.4 litres in 24 h.
Which of the following statements best fits with her condition?

Psychiatric drugs are unlikely to have played a part in her condition

Urine osmolality rising to > 305 mOsmol/l 4 h after desmopressin in the water Your
deprivation test is a positive result for cranial diabetes insipidus, it is unlikely to answer
rise in this case

Nephrogenic diabetes insipidus usually has a dominant pattern of inheritance

Desmopressin in addition to her usual drugs is likely to be effective

Indometacin is of no value

This patient has diabetes insipidus, probably renal in origin related to the use of lithium as
treatment for manic depression. Decompensated diabetes insipidus has features of dehydration
(as in this case), with an inappropriately low urine osmolality. Obviously diabetes mellitus needs
excluding as a cause of polyuria before a diagnosis of diabetes insipidus can be made.
Psychogenic polydipsia is ruled out by the water deprivation test, with urine osmolality rising
appropriately in response to the lack of oral intake. The urine remains dilute in cases of
nephrogenic diabetes insipidus, despite a rising plasma osmolality and urine failing to
concentrate after desmopressin. Desmopressin is unlikely to be effective in this case,

indometacin may be effective. However, if possible, the lithium should be withdrawn in


consultation with a psychiatrist, and another drug such as lamotrigine substituted. Nephrogenic
diabetes insipidus has an autosomal or X-linked recessive pattern of inheritance, whereas cranial
diabetes insipidus is inherited only with an autosomal-dominant pattern.
45. A 62-year-old man presents with bony pain that has been present for some
months, particularly affecting his left femur, pelvis and lower back. Blood
testing reveals a normal serum calcium level, but a raised alkaline
phosphatase. X-rays of the femur and pelvis reveal mixed lytic and sclerotic
change, with accentuated trabecular markings. Chest X-ray is normal.
What is the likely diagnosis?

Secondary carcinoma

Multiple myeloma

Hyperparathyroidism

Hypoparathyroidism

Pagets disease

Your answer

Pagets disease is thought to be present in 2% of the population above 55 years of age, with
90% being asymptomatic. It is more common in Caucasian populations and rarer in populations
of African descent. There are said to be three phases associated with the time course of
radiological changes seen in Pagets disease. Early disease is said to be primarily lytic, then
there are mixed lytic and sclerotic changes, progressing later to primarily sclerotic change with
increasing bony thickening. Goals of treatment are to normalise bone turnover, maintain the
alkaline phosphatase level within the normal range, minimise symptoms and prevent long-term
complications. The mainstay of treatment for this has been use of the bisphosphonates. These
are now often given as intermittent iv courses a few weeks apart. Long-term complications
include deafness (in up to 50% of patients with skull-base Pagets disease), and vary rarely
osteogenic sarcoma.
46. A 32-year-old woman presents with amenorrhoea for 6 months. A pregnancy
test is negative. Over the past few months she has occasionally been leaking
milk, and presents now as this has occurred more and more during stimulation
and intercourse and she is becoming distressed by it. Thyroid function testing
is normal. She is on no medication. Her serum prolactin level is 2400 mU/l and
a CT scan of the pituitary is unremarkable.
Which of the following best fits her condition?

She is likely to have a macroprolactinoma

She should be observed for 12 months

Cabergoline is effective therapy

Your answer

Surgery is the best option here

A visual field defect is likely

She most likely has a microprolactinoma, which would appear as a hypodense area on MRI
scanning. Serum prolactin is often in the range 1500 mU/l to 3000 mU/l in patients with
microadenomas; levels are usually above 3000 mU/l in those with macroadenomas. Although
surgery in conjunction with dopamine-agonist therapy is the treatment of choice for patients
with macroadenomas, microadenomas often respond well to cabergoline (83% normalisation of
prolactin). A visual field defect is unlikely. Observation in these patients is not usually preferable
to normalisation of prolactin levels with dopamine agonists; if this observational approach is
taken then adequate sex-hormone replacement should be undertaken. Of course, normalisation
of prolactin may restore fertility, and the patient should be warned of this possibility.
47. A 55-year-old woman presents with her husband to the endocrine clinic. She
has distressing symptoms of sweating, and her husband noticed increased
prominence of her jaw when he was archiving photos from recent years. Last
year she was diagnosed with type-2 diabetes. Other past history of note is
that she has recently been operated on for carpal tunnel syndrome.
Which of the following most likely to fit with her condition?

Random growth-hormone level is likely to be < 1 mU/l

Your answer

IGF-1 levels are likely to be normal

Growth-hormone levels are likely to remain above 2 mU/l after a 75-g


glucose load

Correct
answer

1-25-OH vitamin D level is invariably normal

Hypertension coexists in 20% of patients with this condition

The diagnosis here is acromegaly. A random growth-hormone level of < 1 mU/l excludes the
diagnosis, growth-hormone levels usually remain above 2 mU/l after an oral glucose tolerance
test. There are increased levels of 1, 25-OH vitamin D in some patients. IGF-1 is invariably
elevated in patients with acromegaly. Sweating is present in more than 80% of cases,
hypertension in 40%, impaired glucose tolerance in 40% and frank type-2 diabetes in 20%.
Coarse facial features including prognathism are often noticed by patients on comparing old
photographs, enlargement of the hands and feet and soft tissue swellings are also common
features. Growth hormone-secreting pituitary tumours are usually identified on MRI scanning
and trans-sphenoidal resection is the treatment of choice.

48. A 32-year-old woman presents with extreme lethargy a couple of weeks after
the birth of her third child by emergency caesarean section. The reason for the
caesarean section was severe blood loss and hypotension. She complained to
the health visitor of increasing problems some 7 days earlier, but was told that
this was to be expected after the birth of her child. On admission via casualty
she was noted to have a sodium concentration of 127 mmol/l, a potassium
concentration of 5.0 mmol/l and a urea of 12 mmol/l.
What is the likely diagnosis?

Sheehans syndrome

Your answer

Hypothyroidism

Primary adrenal failure

Postnatal depression

Dehydration

She has suffered a period of hypotension and blood loss associated with her emergency
caesarean section. This has resulted in pituitary infarction, and she presents now with
symptoms of hypoadrenalism, which include hyponatraemia and potassium at the upper limit of
normal. She, of course, requires fluid rehydration and emergency steroid replacement with iv
hydrocortisone. She will also have pituitary-dependent hypothyroidism and require thyroxine
replacement. Restoration of fertility is more difficult, pulsed delivery of pituitary sex-axis
hormones is usually required. Postnatal depression and simple dehydration are somewhat
unlikely with this set of blood results.
49. A 25-year-old overweight woman presents with hirsutism and
oligomenorrhoea. She has been unable to conceive for 18 months. The
adrenals appear normal on ultrasound scanning, but an ovarian ultrasound
scan reveals numerous small cysts in both ovaries.
Which of the following is likely to fit best with her diagnosis?

The LH/FSH ratio is likely to be normal

Sex hormone-binding globulin is low in 50% of sufferers with this


condition

Testosterone levels are usually normal

Fertility is usually unaffected by this condition

Your answer

DHEAS is usually normal or low

This is polycystic ovarian syndrome (PCOS). The prevalence of PCOS is estimated to range from
8 to 22% of women. Oligo/amenorrhoea is present in 70%, hirsutism in 60%, obesity in 35%
and infertility in 30%. LH is usually markedly raised, as is the LH/FSH ratio. Sex hormonebinding globulin is low in 50% of sufferers. Due primarily to hyperinsulinaemia. Testosterone
and DHEAS (dehydroepiandrosterone) levels are also usually raised. Up to 40% of women with
PCOS may have impaired glucose tolerance, and up to 10% frank type-2 diabetes mellitus. The
insulin-resistant state is also associated with dyslipidaemia. Management is usually with weight
loss and lifestyle advice. Metformin is a useful treatment for insulin resistance and may help
restore fertility, although clomifene may also be required for ovulation induction. Local creams
and electrolysis may have a useful role to play if the primary complaint is one of hirsutism.
50. A 38-year-old woman presents to the clinic with difficult to treat hypertension.
She is on two agents and currently has a BP of 155/95 mmHg. She has noted
that her face has become more rounded over the years and she is having
increasing trouble with both acne and hirsutism. Fasting blood glucose testing
has revealed impaired glucose tolerance. There has also been increasing
trouble with abdominal obesity and she has noticed some purple stretchmarks
appearing around her abdomen.
What is the most likely diagnosis?

Phaeochromocytoma

Multiple endocrine neoplasia

Essential hypertension

Simple obesity

Cushings syndrome

Your answer

Cushings syndrome is associated with a round, plethoric, facial appearance and weight gain in
particular truncal obesity, buffalo hump and supraclavicular fat pads, skin fragility, proximal
muscle weakness, mood disturbance, menstrual disturbance and reduced libido. Hypertension is
present in more than 50% of patients, impaired glucose tolerance in 30%. Osteopenia and
osteoporosis, and premature vascular disease are an inevitable consequence in untreated
Cushings. The annual incidence is approximately 2 per million and the disease is commoner in
women. The disease results from hypercortisolaemia and the cause in 68% of cases is a
pituitary adenoma producing ACTH. Ectopic ACTH production is the cause in 12%, adrenal
adenoma in 10% and adrenal carcinoma in 8%. Diagnosis is made on the results of the 24-h
urinary free-cortisol assay. Differentiation as to the cause is carried out with the
dexamethasone-suppression test and selective venous sampling in cases when a discrete
tumour cannot be identified on contrast-CT scanning.

51. A 24-year-old woman presents with 13 months of amenorrhoea. For the past
few months she has been experiencing hot flushes, night sweats, mood
changes and pain on intercourse. FSH has been > 40 mIU/l on two separate
occasions, and her serum estradiol level is low. TSH is normal. Fasting blood
glucose is normal. Pregnancy test is negative.
What is the most likely diagnosis?

Premature ovarian failure

Your answer

Polycystic ovarian syndrome (PCOS)

Androgen-secreting adrenal tumour

Pituitary failure

Thyrotoxicosis

The blood picture of elevated FSH, low estradiol and prolonged amenorrhoea fits the picture.
The normal TSH rules out thyrotoxicosis, and the raised FSH makes PCOS unlikely and this does
not fit the picture of an androgen-secreting adrenal tumour. Autoimmune disease is responsible
for 20% of cases of premature ovarian failure, and is prevalent in 10% of women with Addisons
disease and 25% of women with autoimmune thyroid disease. Spontaneous recovery of fertility
is unlikely, and put at only 5%.
52. A 44-year-old woman has attended A&E on a number of occasions this year
because of renal tract stones. She has also suffered depression during the past
year or two. She is found to have a serum calcium of 3.10 mmol/l (2.42.6),
creatinine of 138 mol/l and albumin of 40 g/l.
What is her most likely underlying diagnosis?

Hyperparathyroidism

Familial hypercalcaemic hypocalciuria

Multiple endocrine neoplasia

Hypoparathyroidism

Pseudohypoparathyroidism

Your answer

This woman most likely has hyperparathyroidism. Hyperparathyroidism is a feature in 95% of


patients with MEN-1 (multiple endocrine neoplasia type 1) and may coexist with prolactin- or
growth hormone-producing pituitary adenomas, pancreatic islet cell tumours, non-functioning
adrenal adenomas or thyroid adenomas. As part of the MEN-2 syndrome, there is an association
with phaeochromocytoma, Cushings syndrome and medullary carcinoma of the thyroid. Short
metacarpals (usually the fourth or fifth) are associated with pseudohypoparathyroidism, which,
in turn, is associated with hypocalcaemia due to an abnormality of the G-protein receptor for
parathyroid hormone. In the case of parathyroid hyperplasia or parathyroid adenoma, a surgical
solution is the best option where symptoms such as renal stones have begun to appear.
53. A 40-year-old man presents with a serum calcium concentration of 3.05
mmol/l, and urinary calcium excretion of 12 mg/24 h. There is no history of
renal stones, pancreatitis, depression or any prior illness. He was referred by a
particularly zealous GP and underwent parathyroidectomy.
Which of the following is true of the disease which underlies this man's case history?

He has hyperparathyroidism that is likely to have been cured by the


surgery

Urinary calcium excretion is increased

Inheritance is autosomal dominant

Your answer

Correct
answer

Acute pancreatitis is commonly associated with the condition

The body has an increased ability to sense raised calcium levels

This is not hyperparathyroidism. This is familial hypocalciuric hypercalcaemia. It has an


autosomal-dominant pattern of inheritance with virtually complete penetrance. The
heterozygotic state is commonly asymptomatic, but homozygotes present with severe
hypercalcaemia soon after birth and require parathyroidectomy. Occasionally a subgroup of
adult heterozygotes does present with recurrent pancreatitis and may require
parathyroidectomy. The mutation causes a reduced ability for the calcium sensor to detect
hypercalcaemia, so the body tolerates levels of serum calcium that would usually be said to be
outside the normal range.
54. A 51-year-old woman presents to her GP with polyuria, tiredness and a
random plasma glucose level of 13.0 mmol/l. According to the ADA criteria
what should happen next?

She should be reassured that the result is normal

She may have diabetes mellitus and requires a fasting blood test the
following day to confirm the diagnosis

Your
answer

She probably has impaired glucose tolerance and should undergo a 2-h
glucose tolerance test

A 2-h glucose tolerance test plasma glucose level of 10.5 mmol/l would
confirm diabetes mellitus

A fasting plasma glucose of 7.2 mmol/l the following day would suggest
impaired fasting glucose

The American Diabetes Association (ADA) criteria were developed in 1997, with a suggestion
that a glucose tolerance test should be avoided, if possible, to make diagnosis simpler and
quicker. Three methods of classification of diabetes mellitus were developed: symptoms and a
random plasma glucose of greater than or equal to 11.1 mmol/l; fasting plasma glucose of
greater than or equal to 7.0 mmol/l; and 75-g oral glucose tolerance test 2-h glucose of greater
than or equal to 11.1 mmol/l. These diagnostic criteria still stand the test of time today. Ideally
random raised fasting plasma glucose should be confirmed with a fasting sample. Impaired
fasting glucose (analogous to impaired glucose tolerance), is defined as a plasma glucose level
of greater than or equal to 5.6 mmol/l and less than 7.0 mmol/l.
55. A 26-year-old woman has been recently diagnosed with type-1 diabetes. She
has read a great deal about the prognosis of renal disease in type-1 diabetes
and has a number of questions to ask.
Which of the following statements best describes the renal disease in patients
with type-1 diabetes?

Microalbuminuria usually occurs within 2 years of the diagnosis of type-1


diabetes

Peak incidence of frank albuminuria is 17 years after the diagnosis of type- Correct
1 diabetes
answer

After the serum creatinine level reaches 200 mol/l, a fall in GFR of 0.5
ml/min per month might be expected

End-stage renal failure usually occurs within 5 years of the onset of


albuminuria

Your
answer

Nephropathy have a 10 times higher mortality rate than age- and sexmatched controls

In type-1 diabetes microalbuminuria usually occurs 515 years after the diagnosis of diabetes
mellitus. The peak incidence of frank albuminuria is around 17 years after diagnosis. It is
reasonable to expect a fall in the glomerular filtration rate (GFR) of 1 ml/min per month once
the serum creatinine level has reached around 200 mol/traditionally, end-stage renal failure is
said to occur some 710 years after a diagnosis of albuminuria, but it is hoped that improved
management of hyperglycaemia and hypertension may stretch this period a little. Unfortunately,

nephropathy carries a 20100 times higher mortality than that for age- and sex-matched
controls: 40% of deaths of patients with nephropathy are due to cardiovascular disease.
56. A 70-year-old man is brought unconscious to the emergency department; his
blood sugar level is 70 mmol/l. There is no evidence of ketoacidosis. His chest
X-ray reveals evidence of left-sided consolidation.
What is the most likely diagnosis?

Simple pneumonia

A complication of type-1 diabetes

A complication of type-2 diabetes

Your answer

Stroke

Glucagonoma

He is most likely to be suffering from hyperosmolar non-ketotic coma, which is associated with
type-2 diabetes and with coexistent infection. The common presentation is often with blood
glucose levels of over 50 mmol/l, a markedly raised serum osmolality and a decreased conscious
level, without ketosis. Management is with iv insulin and isotonic saline infusion, although the
blood glucose level often drops quickly after insulin is initiated. Hyperosmolar coma may be the
initial presentation of diabetes mellitus, and two-thirds of sufferers present with type-2 diabetes
mellitus for the first time. Mortality is high, particularly in elderly sufferers. Glucagonoma is
vanishingly rare, with an annual incidence of 1/20 million in the UK.
57. A 65-year-old obese woman who has had type-2 diabetes for 4 years presents
to the clinic for her annual review. Her current medication is metformin 2
g/day. You decide to add in gliclazide 80 mg po bd because of poor control as
her Hb A1C is 8.5%.
Which of the following best describes sulphonylureas?

In the UKPDS study they demonstrated no effects on microvascular


outcome

In the UKPDS study they demonstrated no effect on macrovascular


outcome

In the UKPDS study they demonstrated positive effects on


macrovascular outcome

Your answer

Correct
answer

Mild hypoglycaemia in response to sulphonylurea therapy is rare

Gliclazide has positive effects on insulin resistance

Although sulphonylurea therapy was proven by the UKPDS study to have microvascular benefits,
no benefit on macrovascular outcome (cardiovascular disease) was demonstrated: this was, of
course, in contrast to metformin. Sulphonylureas work by stimulating insulin release from
pancreatic beta cells and are associated with mild hypoglycaemia in up to 16% of users,
although hypoglycaemia may be less frequent when short-acting compounds are used. Certain
groups of people (the elderly and those with renal impairment) may be at a higher risk of
hypoglycaemia. Sulphonylureas have no effect on insulin resistance.
58. A 56-year-old, highly insulin resistant, type-2 diabetes sufferer has been
taking 200 units total daily dose of SC insulin per day. His weight is increasing
and his control worsening, with an Hb A1C of 9.1%. You add in pioglitazone 30
mg to his insulin. Some 4 weeks later he presents to the emergency
department in heart failure.
Which of the following statements best describes glitazone therapy?

Glitazones cause heart failure by exerting a directly toxic effect on the


myocardium

Your
answer

Glitazones have no more peripheral insulin-sensitising effects than


metformin

There is evidence that all three agents in the glitazone class (troglitazone,
pioglitazone and rosiglitazone) have similar profiles of hepatotoxicity

Heart failure may be precipitated in some patients taking glitazones due to


fluid retention

Correct
answer

Glitazones act primarily at the site of the PPAR- receptor

There are currently two licensed glitazones (insulin sensitisers) world-wide, rosiglitazone and
pioglitazone. Both drugs act at the site of the PPAR- receptor, promoting the transcription of
genes predominantly linked to fatty acid metabolism. Fibrates, and some newer dual agonists
currently in development, have activity at the PPAR- receptor site. Fluid retention may be
precipitated by these drugs, which may (in less than 1% of users) precipitate heart failure.
Experimental evidence has demonstrated that these drugs actually improve myocardial blood
flow and glucose utilisation. Metformin acts primarily by affecting hepatic insulin resistance,
whereas the glitazones act by improving insulin sensitivity and peripheral glucose uptake in
skeletal muscle and fat. Troglitazone has been withdrawn due to liver side-effects.

59. A 17-year-old young woman presents to the emergency department with


blood glucose of 29 mmol/l. She is known to have type-1 diabetes. Her pH is
7.12 with serum bicarbonate of 11 mmol/l. There is ketonuria.
Which of the following statements best fits the predisposing factors involved in DKA?

Myocardial infarction may be the precipitating factor in up to 5% of


cases of DKA

Infection may be the precipitating cause in 60% of cases of DKA

The patient is not previously known to have diabetes in 30% of DKA


cases

Non-compliance with treatment is the cause in 25% of DKA cases

Your answer

Correct
answer

Inappropriate alterations to insulin are the cause in 20% of DKA cases

The commonest precipitant of diabetic ketoacidosis (DKA) is infection (3040%). This is closely
followed by non-compliance with treatment (25%), alterations to insulin dose (13%), newly
diagnosed diabetes (1020%) and myocardial infarction (< 1%). The condition is characterised
by hyperglycaemia, acidosis and ketonuria. DKA is common, with 9.1% of patients in the
EURODIAB study reporting hospitalisation over the period of a year. DKA is caused by insulin
deficiency and counter-regulatory hormone excess. The mortality rate is 25%, but up to 50%
in more elderly patients.
60. A 36-year-old woman who is 28 weeks pregnant attends the midwife clinic for
a pregnancy check. She is noted to have glycosuria and has a BMI of 30 You
understand her mother has Type-2 diabetes.
What is her correct management plan?

Observe and follow up in 4 weeks' time with repeat urine glucose testing

Carry out a fasting blood glucose; if it is less than 7.0 mmol/l then repeat
the sample in 4 weeks time

Carry out a fasting blood glucose; if it is over 5.5 mmol/l then proceed to
administer a 75-g oral glucose tolerance test

Carry out a fasting blood glucose; it is over 7.0 mmol/l then proceed to
administer a 75-g oral glucose tolerance test

Your
answer

Correct
answer

Proceed directly to administer an oral glucose tolerance test

A glucose tolerance test is required if the patients fasting blood glucose level is over 5.5
mmol/l. Diabetes is defined as a fasting level of > 7.0 mmol/l or > 11.0 mmol/l 2 h after a
glucose tolerance test. Gestational impaired glucose tolerance (IGT) is defined as a fasting blood
glucose level of 6.07.8 mmol/l and/or a 2-h level of 911 mmol/l. Although initial treatment for
gestational diabetes is with dietary advice, insulin is also required in 1030% of cases. Insulin
should be considered for those with fasting levels above 6.0 mmol/l after dietary intervention, or
postprandial sugar levels above 8.0 mmol/l. Untreated gestational diabetes is said to carry a
perinatal mortality rate of 4.46.4%, compared to 0.51.5% in a similarly matched
normoglycaemic population: hence, good glycaemic control is essential for this reason.
61. A 28-year-old mature student nurse is admitted from the emergency
department, where she is on placement, having suffered a collapse. Her blood
sugar was noted to be 1.5 mmol/l and she responded to 50 ml of 50% iv
dextrose. After a second collapse, her insulin level was raised without a
similar rise in c-peptide; her glucose level was 1.9 mmol/l.
What is the most appropriate next investigation?

A 3-day fast in an attempt to precipitate a further attack

Audit of insulin stocks that she has had access to

Your answer

Alcohol history

Chest X-ray to exclude malignancy

Discharge the patient

The clue is the fact that her insulin level was raised without a corresponding rise in c-peptide.
This essentially rules out insulinoma, and suggests that the patient is either injecting excess
insulin herself or is taking sulphonylureas. In fact, on further questioning her aunt has type-2
diabetes; she has been struggling with her studies and taking sulphonylureas to precipitate the
attacks as a cry for help. Standard work-up for hypoglycaemia should include laboratory blood
glucose (not test strips) measurement, liver function tests to rule out significant liver
dysfunction, blood alcohol and alcohol history, cortisol Synacthen testing, insulin and cpeptide levels taken during an attack as well as a chest X-ray to exclude occult malignancy.
62. A 68-year-old man attends his GP on the insistence of his wife because his left
forefoot has become increasingly unstable and abnormally shaped with bony
swelling. He has a long history of diabetes mellitus and his control has been
erratic, with Hb A1C in the range of 910%.
Which of the following pieces of information best fits the pathology or management of
Charcots foot?

Tight glucose control is unlikely to be of benefit in this condition

Reduction of oedema is unimportant with respect to preventing deterioration

Orthotic shoes are unnecessary in the management of this condition

Plain radiography is always abnormal, even early in this condition

The pathology of this condition is thought to be due to sympathetic


dysfunction, excessive blood flow to the joint and osteoclast activity

Your
answer

This man has a Charcot left foot. This is a rare condition now, with the average general hospital
clinic seeing a maximum of 10 Charcots patients. Chronic untreated Charcots foot results in
either rocker-bottom foot due to downward displacement and subluxation of the tarsus, or
medial convexity due to talonavicular joint displacement or dislocation of the tarsometatarsal
joints. Management involves tight glycaemic control, reduction of oedema and orthotic and
chiropody involvement. Intravenous bisphosphonate treatment may be of benefit in some
patients. Although plain radiographs may be normal in the early stages of the disease, they later
show joint destruction, osteolysis, joint reorganisation and subluxation.
63. A 56-year-old patient on gliclazide for his type-2 diabetes presents with an
acute, central, crushing chest pain. He is diagnosed with myocardial infarction.
Which of the following best fits the outcome or management of myocardial infarction
associated with type-2 diabetes?

The mortality rate from myocardial infarction in patients with type-2


diabetes is the same as that for non-diabetics

Intravenous insulin followed by sc insulin after MI reduces mortality by


11% at 3.5 years compared to controls

Your
answer

Use of ACE inhibition after MI improves the 6-week mortality rate by 50%

Statins should always be started unless they are contra-indicated

Correct
answer

Blood pressure target should be 150/80 mmHg

Diabetics may be up to twice as likely to die from myocardial infarction as non-diabetics, and
are more likely to suffer an MI in the first place. The DIGAMI study used iv insulin for 24 h

followed by sc insulin for patients who had suffered an MI. Risk reductions equalled 7.5% at 1
year and 11% at 3.5 years compared to controls. There is now significant doubt about this
result as the study was repeated in the DIGAMI-2 study. This showed no advantage in terms of
cardiac outcomes for patients treated for a prolonged period with insulin. Use of ACE inhibitors
(GISSI-3 Diabetic Subgroup Study) is associated with a 30% relative-risk reduction in the 6week mortality rate (8.7% vs. 12.4%). Blood pressure reduction should be aggressively
targeted, aiming for 130/80 mmHg, and all patients with diabetes who are over 40 years of age
should be started on statin treatment unless there is a contraindication, (Joint British Societies
Guidelines).
64. A 51-year-old patient on metformin and gliclazide for his type-2 diabetes
presents for review. He has a BMI of 31 and his blood pressure is 158/92
mmHg.
When considering hypertension in type-2 diabetes, which of the following statements
best fits the condition?

Development of hypertension is strongly linked to a genetically inherited


variant of red cell sodiumlithium counter-transport activity

Good hypertensive control has a weaker effect on macrovascular outcomes


than good tight glucose control

Hypertension in type-2 diabetes is primarily associated with


hyperinsulinaemia and insulin resistance

Your
answer

Hypertension affects up to 20% of patients with type-2 diabetes

Thiazide diuretics are the treatment of choice in patients with hypertension


and microalbuminuria

Hyperinsulinaemia directly stimulates sympathetic nervous system activity, increasing renal


sodium reabsorption and promoting vascular smooth muscle proliferation. Insulin resistance is
also associated with the reduced activity of nitric oxide synthase, leading to reduced
vasodilatation in response to vascular stress. Red cell sodiumlithium counter-transport activity
abnormalities are associated with the early development of hypertension in type-1 diabetes.
Between 30 and 50% of Caucasians with type-2 diabetes have a systolic blood pressure above
160 mmHg. ACE inhibitors are the treatment of choice for diabetic hypertension, particularly
where microalbuminuria coexists. Thiazide diuretics and -blockers may worsen glycaemic
control.
65. A 52-year-old man is sent by his GP for an urgent review. He has been
maintained on metformin and gliclazide for his type-2 diabetes. For the past
week or so he has had feelings of severe aching pain and paraesthesias in his
upper legs. He has felt off his food during the past week and has begun
losing weight. There is also proximal muscle weakness.
Which of the following best fits diabetic amyotrophy?

75% of patients recover fully from this condition

Transference to insulin therapy is the mainstay of treatment

Correct answer

Recovery from this condition usually takes over 1 year

The condition is much more common in type-1 diabetes

Weight loss is unlikely to be related to the diagnosis

Your answer

Diabetic amyotrophy is said to occur most commonly in men in their fifties with type-2 diabetes
treated with oral hypoglycaemic agents. It is a mixed motor and sensory proximal neuropathy
said to cause severe pain, which is responsible for anorexia and weight loss. Some 50% of
patients recover fully from this condition, usually within 34 months. The mainstay of treatment
is supportive care and transference to insulin therapy.
66. A 63-year-old man presents to the diabetes clinic for review. He attends with
his wife who has accused him of a lack of interest in her as he is no longer
able to sustain an erection.
Which of the following statements is most strongly associated with impotence in type2 diabetes?

Libido is often also affected in addition to physical pathology

Estimates say that 20% of men with diabetes mellitus of more than 6
years duration are affected

Your answer

Other autonomic neuropathy is unlikely to be present

Penile pain is a common association

b-Blockers and thiazide diuretics may exacerbate the problem

Correct
answer

Libido is rarely affected, which makes the condition all the more frustrating for affected men.
Between 30 and 50% of men with diabetes of more than 6 years duration are affected. There is
an association with other autonomic neuropathies such as postural hypotension and
gastroparesis. The condition is not usually associated with pain, and may be exacerbated by
both -blockers and thiazide diuretics. PDE-5 inhibitors are now available as oral therapies for

impotence, but these are often required at high dose. Alprostadil and vacuum devices are
alternative methods. Estimates put successful treatment at only around 70%.
67. A 54-year-old woman presents to the diabetes clinic for review. She has
suffered from diabetes mellitus for many years and has now progressed from
sulphonylurea treatment to insulin therapy. A past history of gallstones is
noted and she failed to tolerate metformin due to chronic diarrhoea. She has
been slowly losing weight over the past few months and has mild anaemia.
Ultrasound scan of her liver reveals a suggestion of a number of small
metastases within the liver.
When considering that this woman has a primary pancreatic tumour, which of the
following statements best fits the condition?

Somatostatin, ACTH and calcitonin may all be raised

Correct
answer

Contrast spiral-CT scanning is effective in demonstrating the primary


tumour in 90% of cases

Your answer

Curative surgery is possible in the majority of cases

There is almost always an association with MEN-1

Most tumours are detected because of the clinical syndrome

The history is suggestive of a somatostatinoma, the annual incidence of which is estimated at 1


in 40 million. There is a 95% association with impaired glucose tolerance (IGT) or diabetes
mellitus, 68% with gallstones, weight loss 25% and anaemia 14%, there is also an association
with diarrhoea. The tumours are often multisecretory and ACTH and calcitonin levels may be
raised in addition to that of somatostatin. There is an association with MEN-1 in some (7%).
Many tumours may occur undetected, and not causing the somatostatinoma syndrome. Contrast
spiral-CT scanning is effective in detecting the primary tumour in only 50% of cases;
radiolabelled octreotide or endoscopic ultrasound scanning may often be required. Although
surgical cure is rarely possible due to the presence of metastases, hepatic embolisation may be
helpful for symptom control.
68. A 52-year-old woman, diagnosed with type-2 diabetes mellitus and losing
weight, is referred for an opinion; her GP is thinking about insulin therapy. A
normochromic, normocytic anaemia is noted. On examination she has angular
stomatitis and a well-demarcated erythematous rash in her groin, which
extends to her lower limbs, buttocks and perineum.
What is the next step in her management?

Refer to nursing colleagues for conversion to insulin

Try high-dose sulphonylurea therapy

Observe and see again in 6 months

Measure plasma glucagon levels

Correct answer

Measure plasma somatostatin levels

Your answer

There is a suggestion that this patient has a glucagonoma, 90% of which are associated with
the characteristic skin rash necrolytic migratory erythema. The annual incidence of
glucagonoma is estimated at 1 in 20 million. Over 70% of glucagonomas are malignant, but,
because of their indolent presentation, the diagnosis is often overlooked for many years. Due to
insulin antagonism over 90% are associated with impaired glucose tolerance, and eventually
weight loss due to protein catabolism. Surgical cure rate is as low as 5%, due to metastases
many years prior to presentation. Octreotide improves the skin rash but may have a detrimental
effect on glucose control; combination chemotherapy with streptazocin and 5-fluoruracil (5-FU)
gives good palliative results, as does hepatic embolisation.
69. A 52-year-old woman is referred for opinion, she has been diagnosed with
type-2 diabetes mellitus and is losing weight, her GP is thinking about insulin
therapy. A normochromic, normocytic anaemia is noted. On examination she
has angular stomatitis and a well-demarcated erythematous rash in her groin,
extending to her lower limbs, buttocks and perineum.
What is the underlying diagnosis?

Poorly controlled type-2 diabetes

Latent type-1 diabetes

Somatostatinoma

Cushings disease

Glucagonoma

Your answer

The skin rash, necrolytic migratory erythema, glucose intolerance, a normochromic, normocytic
anaemia, weight loss and angular stomatitis are all highly suggestive of glucagonoma. The
annual incidence of glucagonoma is estimated at 1/20 million. Over 70% of glucagonomas are
malignant, but because of their indolent presentation, the diagnosis is often overlooked for
many years. Due to insulin antagonism over 90% are associated with impaired glucose
tolerance, and eventually weight loss due to protein catabolism. Surgical cure rate is as low as
5%, as metastases occur many years prior to presentation. Octreotide improves the skin rash

but may have a detrimental effect on glucose control; combination chemotherapy with
streptazocin and 5-fluoruracil (5-FU) gives good palliative results, as does hepatic embolisation.
70. A 36-year-old man is referred after his first myocardial infarction for your
opinion. He has a total cholesterol level of 9.5 mmol/l, with normal
triglycerides. You note the presence of corneal arcus and tendon xanthomas.
Which of the following genetic factors is not likely to be a cause of the
hypercholesterolaemia?

An excess of LDL receptors

Your answer

Not producing any LDL receptors

Failure of the LDL receptors to move to the cell surface

Abnormal receptor-binding to LDL

Inability to internalise LDL for metabolism

Familial hypercholesterolaemia is associated with an autosomal-dominant mutation on the short


arm of chromosome 19, thereby reducing the number of high-affinity, low-density lipoprotein
(LDL) receptors by up to 50%. Over 300 mutations have been described so far and they may be
a combination of any of options BE. Patients present with premature coronary heart disease
(CHD) and have a standardised mortality ratio of up to nine times that of age- and sex-matched
controls. Homozygotes present with CHD in childhood, heterozygotes present with CHD at the
age of 30+ years.
71. A 31-year-old man presents with acute pancreatitis. This is the third occasion
he has presented in the last 5 years. You follow him up in clinic, and find he
has a markedly elevated triglyceride fraction and hypercholesterolaemia, but
with HDL and LDL cholesterol within the normal range.
Which of the following statements best fits his abnormal lipid picture?

Usually improved by thiazide diuretics

Usually improved by glucocorticoids

Usually improved by alcohol

Has an autosomal-recessive mode of inheritance

Your answer

Affects up to 1 in 300 people

Correct answer

This is familial hypertriglyceridaemia, an autosomal-dominant condition affecting 1 in 300


people, associated with raised very-low-density lipoproteins (VLDL) and triglyceride levels. It
may be exacerbated by alcohol, glucocorticoids and thiazide diuretics. Avoidance of alcohol is
advised. Clinical features include eruptive xanthomas and lipaemia retinalis.
72. A 61-year-old, non-smoking woman with no previous cardiac history presents
from her GP at the cardiac risk-factor clinic. Her total cholesterol is 9.0
mmol/l. She is overweight and has sleep apnoea. On examination you notice
her skin is particularly dry and there appears to be some evidence of hair loss.
What is the next step in her management?

Screening of family members for hypercholesterolaemia

Lifestyle advice and reassurance

Start high-dose statin therapy

Check her TSH

Your answer

Check her fasting blood glucose

The suspicion with the history of dry skin, hair loss, obesity and sleep apnoea is that she has
hypothyroidism. This would be the cause of her secondary hypercholesterolaemia. Frank
hypothyroidism is said to occur in 4% of patients with dyslipidaemias, and a further 10% of
patients with dyslipidaemia have a raised TSH level, but with a free T 4 still within the normal
range. Total cholesterol often improves somewhat with thyroxine therapy, but statins may be
required in addition.
73. You review a 67-year-old man in the lipid clinic. He is taking pravastatin 40
mg/day. Which of the following drugs should be used with caution?

Warfarin

Erythromycin

Digoxin

Antacids

Correct answer

Antifungals

Your answer

Warfarin and digoxin have well-recognised interactions with atorvastatin and simvastatin.
Antacid and antifungal agents are known to interact with atorvastatin. Erythromycin has known
interactions with simvastatin, fluvastatin, atorvastatin and pravastatin. All statins interact with
ciclosporin and nicotinic acid and should be used with caution in patients on fibrate therapy as
the risk of rhabdomyolysis may be increased.
74. The presence of galactorrhoea is MOST suggestive of which one of the
following conditions?

Turners syndrome

Polycystic ovary disease

Hypothyroidism

Your answer

Sheehans syndrome

Bromocriptine therapy
Galactorrhoea is nonpuerperial expression of milk. Hyperprolactinaemia causes galactorrhoea
and amenorrhoea, which may result from a prolactin secreting pituitary tumour or hyperplasia.
Hypothyroidism is a cause of raised prolactin levels. In acromegaly one-third of patients have
mild elevation of prolactin levels resulting in galactorrhoea, amenorrhoea and decreased libido.
Similar elevation of prolactin levels occurs in a small percentage of patients with primary
hypothyroidism. Sheehans syndrome is a primary hypopituitarism due to ischaemic necrosis of
the pituitary gland caused by postpartum haemorrhage; it is characterised by failure of
postpartum lactation and failure to resume normal cyclic menstruation. Turners syndrome
(ovarian dysgenesis) causes primary amenorrhoea with poor development of the breast.
Bromocriptine is a dopaminergic agent that has an inhibitory effect on prolactin and is
frequently used to treat hyperprolactinaemia. Galactorrhoea and raised prolactin levels may also
be seen in PCOS.
75. Which one of the following proteins is most likely to be associated with very
high levels of plasma chylomicrons?

Apoprotein E

Your answer

Apoprotein CII

Correct answer

Apoprotein AII

Lipoprotein B

LDL receptor

Dietary triglycerides in cholesterol are packaged by gastrointestinal epithelial cells into large
lipoprotein particles called chylomicrons. After secretion into the intestinal lymph and passage
into the general circulation, chylomicrons bind to the enzyme lipoprotein lipase, which is located
on endothelial surfaces. This enzyme is activated by a protein contained in the chylomicron,
apoprotein CII, liberating free fatty acids and monoglycerides, which then pass through the
endothelial cells and enter adipocyte or muscle cells. Therefore, complete inactivation of either
lipoprotein lipase or apoprotein CII as a result of the inheritance of two defective copies of the
relevant gene results in an accumulation of chylomicrons (type I lipoprotein elevation) owing to
failure of conversion to the chylomicron remnant particle. Patients with familial lipoprotein lipase
deficiency usually present in infancy with recurrent attacks of abdominal pain caused by
pancreatitis. They also have eruptive xanthomas resulting from triglyceride deposition.
Treatment should consist of a low-fat diet that may be supplemented by medium-chain
triglycerides, which are not incorporated into chylomicrons. The absence of functional apoprotein
CII, with consequent failure to activate lipoprotein lipase, presents with a similar phenotype,
although the affected patients are typically detected at a somewhat later age than are patients
with familial lipoprotein lipase deficiency.
76. A 65-year-old woman known to have chronic low back pain notices severe
sharp pain in the left groin after a minor fall and is unable to walk. Left neck of
femur fracture is identified on radiological examination. Routine laboratory
evaluation discloses a serum calcium concentration of 1.9 mmol/l, a serum
phosphorus concentration of 0.68 mmol/l and increased serum alkaline
phosphatase activity. The serum parathyroid hormone level was subsequently
found to be elevated. The most likely diagnosis is?

Primary hyperparathyroidism

Hypervitaminosis D

Pagets disease of bone

Osteoporosis

Your answer

Vitamin D deficiency

Correct answer

The combination of hypocalcaemia and hypophosphataemia points to the diagnosis of


osteomalacia and vitamin D deficiency. Dietary deficiency and malabsorption are common
causes of vitamin D deficiency. Primary hyperparathyroidism and hypervitaminosis D are
associated with hypercalcaemia rather than hypocalcaemia. Pagets disease is associated with
increased risk of fracture and increased serum alkaline phosphatase activity, but the serum

calcium is usually within normal limits. Osteoporosis is the most common cause of fracture of
neck of femur and is not associated with any specific abnormality in the standard bone
biochemistry profile.
77. In glucagonoma the MOST likely associated skin lesion is?

Erythema chronicum migrans

Acanthosis nigricans

Panniculitis

Ichthyosis

Necrolytic migratory erythema

Your answer

Glucagonoma syndrome (diabetes mellitus, weight loss and anaemia) is associated with a
characteristic skin rash (necrolytic migratory erythema) in 75% of cases. The lesion starts as an
indurated erythema at the perineum, face and nose. Within a few days blisters will cover the
surface of the skin which then crust and heal leaving hyperpigmented skin. This process takes
714 days with lesions developing in one area while others are resolving.
78. A 23-year-old woman presents to her GP after the birth of her second child.
She complains of extreme tiredness and a persistent hoarse voice that she is
having problems shaking off. Despite breast-feeding her child she is failing to
lose her pregnancy weight. Thyroid autoantibodies are negative. Her TSH is 12
mU/l, with a free T4 of 5 pmol/l. There is no thyroid tenderness on
examination. Her GP notes that her pulse is only 52 beats per minute.
What diagnosis fits best with this clinical picture?

Hashimotos thyroiditis

Postpartum thyroiditis

Atrophic hypothyroidism

Iodine deficiency

Hyperthyroidism

Your answer

This woman is hypothyroid after the birth of her second child. Her thyroid autoantibodies are
negative and she has no signs of autoimmune disease, which might suggest atrophic
hypothyroidism were they present (eg vitiligo). Iodine deficiency is now rare and tends to occur
in isolated mountain areas. Postpartum thyroiditis is usually transient, and may involve
hyperthyroidism, hypothyroidism or the two in sequence. It is thought to be due to changes in
the immune system after pregnancy and is histologically a lymphocytic thyroiditis. It is usually
self-limiting.
79. A 57-year-old woman presents with a feeling of shortness of breath and
choking on lying down. Some 4 months earlier she had been diagnosed with
atrial fibrillation and was started on aspirin and digoxin by her GP. On
examination her GP could feel goitre. Plain radiography confirmed retrosternal
extension, which was presumed to be contributing to her shortness of breath.
Her TSH level was less than 0.05 mU/l. Thyroid autoantibodies were negative.
What diagnosis best fits with this clinical picture?

Hashimotos thyroiditis

Large, toxic, multinodular goitre

Your answer

Thyroglossal cyst

Thyroid carcinoma

Graves disease

Toxic multinodular goitre usually occurs in women over 55 years of age and is more common
than Graves disease in the elderly. Her atrial fibrillation may well be related to this. Her goitre is
obstructing, with significant retrosternal extension, and surgery is the treatment of choice.
Initially, however, she should be rendered euthyroid with antithyroid drugs such as carbimazole.
80. A 62-year-old man presents for review some 3 months after first being
diagnosed with type-2 diabetes. His BMI is 30. Despite having lost about 7 kg
in weight, his morning blood sugars are still around 9 mmol/l; an Hb A1C check
was 8.9%. He is hypertensive and taking ramipril, his triglycerides are raised
and his HDL cholesterol is low.
Which therapy for his diabetes would be the best initial choice for his
hyperglycaemia?

Glibenclamide

Gliclazide

Rosiglitazone

Metformin

Your answer

Acarbose

The best initial therapy for this man, who clearly has the metabolic syndrome, is metformin.
This drug should be introduced at a dose of 500 mg per day, with a gradual increase in the
dose over a few weeks to around 1.52 g total daily dose (divided into morning and evening
doses).

The UKPDS (United Kingdom Prospective Diabetes Study) showed that for macrovascular risk,
metformin was superior to sulphonylureas or insulin (a statistically significant risk reduction
for myocardial infarction compared to conventional therapy, which was not found in the
sulphonylurea or insulin group). Metformin is a partial insulin sensitiser that works to reduce
hepatic glucose output and also has some anti-inflammatory action, showing positive effects
on plasminogen-activator inhibitor-1 (PAI-1) in particular. PAI-1 is associated with an
increased tendency to blood clotting and may be associated with an increased vascular risk in
sufferers of the metabolic syndrome.

81. A 57-year-old man, with a BMI of 30 and a history of hypertension,


dyslipidaemia and type-2 diabetes presents for review. He has tolerated 2
g/day of metformin well, but 2 years after the initial diagnosis of diabetes his
blood sugars are still too high in the morning on occasions, and a recent HB
A1C was 7.5%. His job entails occasional shift work, during which he is unable
to eat for long periods.
Which would be the most appropriate add-in therapy to his metformin treatment?

Acarbose

Glargine

Pioglitazone

Correct answer

Chlorpropamide

Glibenclamide

Your answer

The choice is between the addition of sulphonylurea therapy, insulin or a glitazone.

Sulphonylurea therapy or insulin may not be appropriate due to his shift work and risk of
hypoglycaemia. In addition, he has features of the insulin resistance (metabolic) syndrome,
and so a glitazone may be the more logical addition.

Rosiglitazone and pioglitazone work by increasing glucose uptake into skeletal muscle and fat.
They bind to the PPAR-gamma nuclear receptor (PPAR, peroxisome proliferator-activated
receptor), which promotes the transcription of a number of enzymes concerned with glucose
and lipid metabolism. Clinical trials indicate that by targeting insulin resistance they appear to
have positive effects on other features of the metabolic syndrome such as blood pressure and
HDL cholesterol. There is some debate around the cardiovascular effects of rosiglitazone.

82. A 56-year-old man with type-2 diabetes presents with background diabetic
retinopathy. His HB A1C has been consistently above 9% for the past 5 years.
Which of the following factors would most worsen prognosis for his retinopathy?

Rapid improvement in blood glucose levels

Your answer

Total cholesterol 5.2 mmol/l

Long-term improvements in blood glucose control

Triglyceride levels of 2.1 mmol/l

Stopping smoking

Rapid improvement in blood glucose levels may be associated with worsening of diabetic eye
disease. Both the Diabetes Control and Complications Trial (DCCT) in type-1 diabetes and the
UKPDS (United Kingdom Prospective Diabetes Study) in type-2 diabetes have demonstrated
the long-term benefits of reducing blood sugar levels in controlling future microvascular
complications. All diabetic patients should be offered yearly eye screening, preferably with a
digital retinal camera. Rapidly deteriorating visual acuity, hard exudates encroaching on the
macula, preproliferative changes or new-vessel formation are all reasons for early referral to
an ophthalmologist.

Diabetic retinopathy has been known for many years to be the leading cause of blindness in
the UK in people of working age, but progression to blindness is now slowing and tailing off
due to improved ophthalmic care. Smoking has no effect on the prognosis of retinopathy.

83. A 45-year-old man is to undergo trans-sphenoidal surgery for resection of a


growth hormone-secreting pituitary adenoma. He asks about the prospect of a
cure and you explain that he will require biochemical monitoring over the next
few months and years to assess this.

Which of the following biochemical tests is the best way to monitor for recurrence?

Blood glucose

Serum cortisol

IGF-1 or growth hormone level

Your answer

Prolactin

Thyroid function testing

The blood glucose level may be a marker of recurrence via the development of diabetes
mellitus, but in itself is not the best way to monitor. The aim of therapy is to keep the growth
hormone level below 5 mU/l or insulin-like growth factor-1 (IGF-1) levels within normal limits.
Growth hormone levels above 5 mU/l are associated with a worse prognosis. Quoted failure
rates for surgery are around 13.3% for tumours of < 1 cm or 11.1% for tumours > 1 cm
confined to the sella.

Treatment failure may be managed with somatostatin analogues or external-beam


radiotherapy. Chronic somatostatin analogue use may be associated with increased gallstones,
and the response to external-beam radiotherapy is often slow, so neither treatment is perfect.

84. A 42-year-old man presents to his GP complaining of decreased libido. He has


also noticed having to shave less frequently. On the few occasions he has tried
to have sex he has failed to maintain his erection to penetration. His visual
field testing is normal and he has no medication history. Serum prolactin
levels are 890 mg/l, growth hormone and thyroid function levels are normal.
What diagnosis fits best with this clinical picture?

Microprolactinoma

Macroprolactinoma

Psychogenic impotence

Hypothyroidism

Acromegaly

Your answer

Men with prolactinoma tend to present substantially later than women, the latter presenting
with cessation of periods and galactorrhoea. Macroprolactinoma is unlikely given his normal
visual fields. Prolactinoma is the most common pituitary tumour; microadenomas are more
common in women, whereas macroadenomas appear more commonly in men. Treatment is
surgical (via the trans-sphenoidal route) or medical (microprolactinomas) using dopamine
agonists such as bromocriptine or cabergoline to suppress prolactin release. Trans-sphenoidal
cure rates are around 5075%. Up to 20% of microprolactinomas resolve during long-term
dopamine agonist treatment.
85. A 19-year-old student is brought to A&E by his flatmates. He had been playing
squash that afternoon, and while resting (after having had a pasta meal),
complained of generalised weakness. He was unable to stand and had to be
carried in by his friends. His potassium level was noted to be 2.6 mmol/l.
Urine screen for diabetic and laxative abuse was normal. Apparently, he has
had similar attacks since his early teenage years. Symptoms were aborted by
potassium chloride.
What type of mutation best fits the underlying pathology of this autosomal-dominant
condition involving intermittent paralysis?

Mutation of a muscle voltage-gated sodium channel

Mutation in a muscle voltage-gated potassium channel

Your answer

Mutation in a renal potassium channel

Mutation in a renal sodium channel

Mutation in a muscle voltage-gated calcium channel

Correct answer

Hypokalaemic periodic paralysis is related to a muscle calcium-channel mutation (CAClN1A3). It


is an autosomal-dominant condition; attacks usually begin in the teenage years and may remit
after around 35 years of age. Potassium is normally below 3 mmol/l during an attack and
symptoms resolve with the administration of potassium chloride. Attacks appear to be
precipitated either by a high carbohydrate meal or by a period of rest after extreme exercise.
Loss of function of muscles concerning speech, bulbar or generalised weakness may occur and
attacks can last for several hours.
86. A 23-year-old woman with type-1 diabetes presents with an unusual lesion on
her shin. It began as a patch of spreading erythema, but now looks yellow and
has begun to ulcerate.
What diagnosis best fits this lesion?

Acanthosis nigricans

Dermatitis herpetiformis

Granuloma annulare

Your answer

Necrobiosis lipoidica

Correct answer

Xanthoma

Necrobiosis lipoidica occurs in patients with type-1 diabetes, beginning as a patch of erythema
that spreads across the shin, begins to yellow and then may ulcerate. Acanthosis nigricans is
associated with insulin resistance and may occur in those with type-2 diabetes and is usually
found in the axillas. Granuloma annulare is normally diffuse in diabetes and occurs as a ring of
papules. Dermatitis herpetiformis is, of course, associated with coeliac disease (gluten
enteropathy).
87. A 71-year-old man with established sarcoidosis presents for review. He suffers
from a degree of pulmonary fibrosis due to previous pulmonary infiltration and
has been taking corticosteroids intermittently. You are asked for a
consultation as his serum calcium concentration is 3.1 mmol/l. His renal
function is normal and parathyroid hormone is just below the lower end of the
normal range.
What is the most likely cause of his hypercalcaemia?

Hyperparathyroidism

Hypoparathyroidism

Increased hydroxylation of Vitamin D

Your answer

Chronic renal failure

Milk-alkali syndrome

In this case, parathyroid hormone is virtually in the normal range and his renal function is
normal, so there is nothing to suggest a diagnosis of the milkalkali syndrome. Hypercalcaemia
is found in 10% of established cases of sarcoid, and may eventually lead to nephrocalcinosis.
The cause is increased 1 a-hydroxylation of vitamin D by sarcoid macrophages, in addition to
that taking place in the kidney.

88. You are asked to review a 54-year-old psychiatric patient by his GP. This
patient has been diagnosed with impaired glucose tolerance. He also has a
history of hypertension, for which he takes ramipril.
Which of the following drug classes is most well known as a cause of impaired
glucose tolerance?

Thiazolidinediones

ACE inhibitors

Atypical antipsychotics

Your answer

Biguanides

Sulphonylureas

Older atypical antipsychotic agents have been implicated as a cause of impaired glucose
tolerance, and are thought to put those taking them at increased risk of type-2 diabetes.

Thiazides and -blockers may also impact negatively on the incidence of new diabetes, and
this combination of antihypertensives is no longer a first choice for patients with a high type-2
diabetes risk.

Ramipril was shown in the HOPE (Healthcare Options Plan Entitlement) study to reduce
progression to type-2 diabetes; this is thought to be mediated by a reduction in insulin
resistance. The LIFE (Longitudinal Interval Follow-up Evaluation) study using losartan has
demonstrated that angiotensin-receptor blockers also probably reduce the incidence of new
cases of type-2 diabetes.

Thiazolidinediones and biguanides work by primarily reducing insulin resistance,


sulphonylureas stimulate insulin release; all three are used as treatments for type-2 diabetes.

89. You are asked by a GP to review a 16-year-old girl who presents with primary
amenorrhoea. She appears on examination to have minimal body hair but
normal breast development. Examination also reveals a blind-ended vagina.
Biochemistry reveals increased LH, normal FSH, raised estradiol and raised
testosterone levels.
Which diagnosis fits best with this history and examination?

Polycystic ovarian syndrome

Turners syndrome

Ashermans syndrome

Testicular feminisation

Your answer

Pregnancy

This picture fits a diagnosis of testicular feminisation, in which androgen receptors are
defective. This girl, although having the external phenotype of a female, will have the XY
karyotype. These individuals are, of course, infertile. Patients with testicular feminisation do
possess testes, and orchidectomy is advised since there is a high risk of testicular malignancy.

Girls with Turners syndrome usually have the typical Turners phenotype of short stature and
webbed neck. Ashermans syndrome is amenorrhoea due to uterine synechiae after infection;
there is no history to suggest this and it can be ruled out on the examination findings.

90. A 27-year-old man presents via his GP for review. He wants to start a
relationship but is concerned about his small phallus. He also has difficulty
becoming aroused. On examination he is slim and there is gynaecomastia.
There is a general paucity of body hair, his penis is small and he has small
testes. Which diagnosis fits best with this history and examination?

Testicular feminisation

Klinefelters syndrome

Your answer

Congenital adrenal hyperplasia

True hermaphroditism

5 -reductase deficiency

This man is likely to have karyotype XXY and has the features of Klinefelters syndrome.
Individuals with testicular feminisation are phenotypic females. Karyotypic females with
congenital adrenal hyperplasia may have clitoromegaly and labial fusion, as well as ambiguous
external genitalia that may appear male; they do not, however, have testes.

True hermaphrodites are vanishingly rare and may be XX/XY or mosaic in karyotype; they
possess both testes and ovaries and usually have male or ambiguous external genitalia.

91. A 54-year-old man presents to the diabetes clinic for review. He has had type1 diabetes for 30 years. Recently he has suffered a number of falls, which he
describes as attacks where he feels faint and loses his footing. He has
suffered from impotence for a number of years and takes antireflux
medication. On examination he has a postural drop of 35 mmHg in his blood
pressure. Which diagnosis fits best with this history and examination?

Diabetic autonomic neuropathy

Your answer

Transient ischaemic attacks

Arrhythmia

Simple fainting

Somatisation disorder

This man has a marked postural drop, features of GI tract neuropathy and impotence. Further
assessment of his cardiovascular system is likely to reveal tachycardia and impaired
cardiovascular response to the Valsalva manoeuvre. Other symptoms of GI tract involvement
may include diarrhoea and intractable vomiting. Bladder involvement may result in atonic
bladder and problems with painless urinary retention and recurrent urinary tract infections.
92. A 42-year-old woman presents with difficult-to-treat hypertension. She is
taking ramipril, atenolol and bendrofluazide yet her blood pressure as
measured in the clinic is still 150/100 mmHg. Her serum potassium
concentration as measured in clinic was 2.9 mmol/l. She has been weaned off
her antihypertensives for a period of 46 weeks; at this time her renin level
was noted to be suppressed and her aldosterone level was above normal.
Abdominal ultrasound suggests enlargement of the right adrenal.
Which of the following diagnoses is most likely to fit with this clinical picture?

Idiopathic hyperaldosteronism

Conns adenoma

Glucocorticoid suppressible hyperaldosteronism

Your answer

Aldosterone-producing carcinoma

Carcinoid syndrome

There is controversy about the commonest cause of hyporeninaemic hypoaldosteronism,


previously 60-80% were said to be related to an adenoma, but the prevailing view now is that
75% may be related to adrenal hyperplasia. Glucocorticoid suppressible hyperaldosteronism
and aldosterone-producing carcinoma make up less than 1% of cases each. Primary
hyperaldosteronism is estimated to be responsible for 12% of cases of hypertension and is
generally asymptomatic. Rarely, there may be muscle cramps associated with hypokalaemia.

To estimate renin and aldosterone levels accurately it may be necessary to wean the patient
off antihypertensive drugs for a period of a few weeks, since most classes of antihypertensives
have effects on the reninangiotensin axis. CT or MRI scanning may be used to confirm the
adenoma.

After removal of the adenoma, blood pressure is normal in 70% of patients at 1 year. At 5
years postsurgery, 50% of patients remain normotensive.

93. An 18-year-old girl is referred by her GP who is concerned that she may have
an underlying endocrine problem. The girl had been taken to the GP by her
mother. She is a good student and has just won a place at university. She
weighs only 38 kg (6 stone) and is 1.78 m (5ft 10 inches) tall. She is
emaciated, her skin is dry and she has excessive growth of lanugo hair. She
has been amenorrhoeic for 9 months. Her cortisol level is elevated, her free T4
is normal. She has an anaemia and associated reduced white cell and platelet
count. Which of the following diagnoses is most likely to fit with this clinical
picture?

Addisons disease

HIV

Occult carcinoma

Hypothyroidism

Anorexia nervosa

Your answer

Anorexia nervosa has a female to male preponderance of 9:1. It is estimated that around 0.5
1% of American women between 15 and 30 years of age have anorexia, and the numbers are

likely to be similar in the UK. The aetiology of anorexia is unknown, but it is likely to be an
interaction between environmental and genetic factors, American studies report rates of
sexual abuse as high as 50% in anorexic females. Despite an association with depressive and
other psychiatric illness, demographic studies show no impairment of cognitive level nor
educational achievement in the group of patients with anorexia nervosa as compared to the
general population.
Indeed such patients may fit the profile of being "high achievers", which is what is being
hinted at in this question.

Typical laboratory tests include decreased FSH, LH, oestrogens and 17-OH steroids. Free T 4
and TSH levels are usually normal. Anaemia with decreased white and platelet cell count may
also occur. There may be metabolic alkalosis, hypocalcaemia, hypokalaemia and
hypomagnesaemia.

Treatment may involve complex psychotherapy for a number of years, and referral to a
specialist in the field is recommended.

94. A 60-year-old woman is sent by A&E for endocrine review. During the past 18
months she has suffered two Colles fractures and a fractured neck of her left
femur. Results of thyroid function testing, serum protein electrophoresis and
serum parathyroid hormone estimation are all normal. Bone densitometry of
the lumbar spine and femoral neck on the non-replaced side reveal a bone
density within the osteoporotic range. Which of the following interventions
would be most appropriate for her?

Observe and repeat the densitometry in 12 months

Initiate bisphosphonate therapy

Correct answer

Initiate calcium and vitamin D therapy

Initiate HRT

Your answer

Initiate calcium supplementation

This woman has idiopathic osteoporosis, with normal thyroid function and parathyroid hormone
assay ruling out hyperthyroidism and hyperparathyroidism. Her osteoporosis is clearly of clinical
significance in that she has suffered two fractures of the distal radius and a fractured neck of a
femur during the past 19 months. Hormone replacement therapy (HRT) would not be advised in
this age group due to the increased cardiovascular and breast cancer risk. Bisphosphonate
therapy, which inhibits osteoclast activity, would be the best choice in this patient.
95. A 58-year-old woman is taking alendronate for osteoporosis. She visits the
clinic and is keen to discuss the mechanism of action of this class of drugs as

she has been studying them on the Internet. Which of the following options
best describes the mode of action of the bisphosphonate class of agents?

Inhibits osteoblast activity

Stimulates osteoblast activity

Stimulates osteoclast activity

Inhibits osteoclast activity

Your answer

Increases the bioavailability of vitamin D

The primary mode of action of the bisphosphonates is inhibition of osteoclast activity. With the
results from large-scale HRT studies suggesting an increased risk of carcinoma or cardiovascular
disease, they have become the first-choice therapy for osteoporosis. They may be given orally,
although some clinicians prefer intermittent intravenous use. This class of drugs may be
associated with an increased risk of oesophageal ulceration. When the class was developed
there was initial concern over prolonged inhibition of bone turnover over a number of years, but
this appears to have been unfounded.
96. A 61-year-old farmer who has long-standing type-1 diabetes is brought to the
clinic by his wife. He has been limping for a while and his wife noticed that his
ankle was rather abnormally shaped after he stepped out of the shower.
Examination of his right ankle reveals a painless joint that is warm and
swollen. There is crepitus and what appears to be palpable bone debris. X-ray
reveals gross joint destruction and apparent dislocation. His CRP and white
count are normal, the joint aspiration fluid shows no microbes and historical
review of HB A1c reveals that it has rarely been below 9%. What is the most
likely diagnosis in this case?

Charcots ankle

Osteomyelitis

Old healed fracture

Rheumatoid arthritis

Osteoarthritis

Your answer

Charcots joint is said to occur in 1 in every 750 patients with diabetes mellitus, although this is
increased to 5 in every 100 in those with proven neuropathy. However, a Charcots joint does
not just occur in those with diabetes but may occur in 2040% of patients with syringomyelia
and 510% of those with long-standing syphilis (although this condition is now very rare).
Management in diabetes includes tight control of blood glucose, appropriate orthotic
intervention, and there is evidence that bisphosphonates may slow the pace of joint destruction.
97. A 32-year-old merchant banker presents for endocrine review. Apart from a
past history of reflux symptoms, her previous medical history is unremarkable.
She gives a history of increasing fatigue, being sometimes unable to leave her
bedroom; she says that she is only able to walk a few steps without feeling
exhausted. She has lost a little weight and gone on long-term sick leave from
her job. She feels unable to concentrate, has headaches, intermittent sore
throats and feels that when she is able to sleep, she awakes unrefreshed. Full
blood count, viscosity, urea and electrolytes, liver function testing, thyroid
function testing and a Synacthen test arranged by her GP have all been
normal. On examination in the clinic there are no abnormal physical findings in
this normal weight, normal height young woman. What diagnosis fits best
with this clinical picture?

Addisons disease

Hypothyroidism

Chronic fatigue syndrome

Your answer

Myasthenia gravis

Occult malignant disease

The prevalence of chronic fatigue syndrome is estimated at 13 per 1000 individuals. The
predominant age at presentation is young adulthood to middle age, and there is a slight female
preponderance. Sufferers often report a short flu-like illness at the point of onset and some
scientists believe that an initial viral trigger may be responsible. Conventional investigation and
physical examination proves normal. Initial hopes centred on the EpsteinBarr virus infection
being that trigger, but concrete proof has not yet been demonstrated. Education, counselling
and goal setting have proved useful in stimulating recovery, and trials have demonstrated
improvements with amitriptyline use, but many patients take years to return to normal life.
98. A 62-year-old man presents with evidence of gynaecomastia. He has been
taking long-term digitalis and warfarin therapy for persistent atrial fibrillation.
In addition, he takes another tablet for heartburn. Results of tests for
androgens, HCG, liver function and thyroid function are all normal.
What is the most likely cause of his gynaecomastia?

Warfarin therapy

Digoxin therapy

Correct answer

Furosemide

Ranitidine

Your answer

Sodium bicarbonate

Digitalis is one of a number of drugs that may be associated with gynaecomastia. Other causes
include oestrogens, cannabis, diamorphine, spironolactone, cimetidine, cyproterone,
gonadotrophins and some cytotoxics. Whilst gynaecomastia is occasionally seen with ranitidine,
this is much less common versus digoxin or cimetidine. Thyrotoxicosis, liver disease, oestrogenproducing tumours, HCG-producing tumours, starvation/refeeding and carcinoma of the breast
are also associated with gynaecomastia. Surgical removal is occasionally carried out in younger
men.
99. A 21-year-old university student presents for review. She is distressed by the
fact that she is overweight and is having to shave or pluck excessive facial
hair. She also notices that she appears to have more generalised body hair
than other women. On further questioning you elicit a history that she can
sometimes miss a couple of menses, but pregnancy testing is consistently
negative. Her mother apparently had similar problems in her youth, took a
while to conceive and now has type-2 diabetes. Testosterone is just outside
the upper limit of normal range, her LH: FSH ratio is increased and prolactin is
normal. Which diagnosis fits best with this womans clinical picture?

Polycystic ovarian syndrome (PCOS)

Your answer

Hyperprolactinaemia

Androgen-secreting tumour

Cushings disease

Type-2 diabetes

PCOS is said to occur in 3% of adult women. Symptoms usually begin around the time of
menarche and the diagnosis is often made in adolescence. Obesity is present in around 40% of
sufferers and there is an increased association with type-2 diabetes due to insulin resistance.
Biochemical abnormalities characteristically include a mildly raised testosterone level and an
increased LH: FSH ratio. Treatment includes weight loss or metformin therapy. Surgical
intervention with wedge ovarian resection may reduce androgen secretion and symptoms. Many
women just require reassurance and effective local treatment for hirsutism. Those who wish to

conceive do so with increased frequency after a trial of metformin therapy, clomifene, or in


some cases gonadotrophins.
100.
A 21-year-old woman presents for review. She is concerned on this
occasion because she has not had a period for 5 months. She is 1.76 m in
height and weighs only 43.7 kg (7 stone). A pregnancy test is negative and
thyroid function testing is normal. Which diagnosis fits best with this womans
clinical picture?

Gonadotrophin deficiency

Weight-related amenorrhoea

Your answer

Hyperprolactinaemia

Primary ovarian failure

Polycystic ovarian syndrome

A woman needs to maintain a minimum body weight for menstruation, and amenorrhoea may
even be seen at weights considered to be at the lower end of the normal range. The biochemical
picture may be indistinguishable from gonadotrophin deficiency. Gaining body weight to above
the 50th centile for height normally results in the restoration of menstruation, but if this cannot
be achieved then oestrogen replacement may be considered. Amenorrhoea is often seen in
ballet dancers who maintain a low weight and carry out periods of extreme physical exercise.
101
. A 75-year-old woman is admitted in an unconscious state. Her daughter
found her on the floor. On examination in casualty she is found to have a core
temperature of 33 C and also to be in left ventricular failure. Her blood glucose level
is 5.7 mmol/l, random cortisol is elevated. By chance you also catch the twice weekly
run of thyroid function testing and her free T4 is 4.4 pmol/l. A CT scan of her brain
reveals no focal lesion and a cursory assessment reveals no gross focal neurology.
Which diagnosis fits best with this womans clinical picture?

Hypoglycaemia

Your answer

Addisons disease

Profound hypothyroidism

Massive stroke

Correct answer

Alcohol excess

This woman has a greatly reduced free T4 concentration, is hypothermic, unconscious and has
evidence of associated heart failure. Mortality associated with this condition used to be as high
as 50%, but with modern intensive care management, survival has improved. T 3 is usually given
via a nasogastric (NG) tube or iv injection at the rate of 2.55 g every 8 h, with conversion to
T4 after the patient regains consciousness. There is a risk of precipitating heart failure if larger
doses are given in the initial period. Other supportive measures include oxygen therapy,
hydrocortisone iv therapy and glucose infusion, but none of these have been effectively proven
in a randomised, controlled trial setting.
102.
A 62-year-old woman undergoes subtotal thyroidectomy for goitre.
Some 12-h postsurgery she calls the duty doctor and is found to be suffering
from pins and needles in her hands and carpopedal spasm. An urgent
corrected calcium level is found to be 2.1 mmol/l. What is the most likely
cause of her hypocalcaemia and what is the long-term prognosis?

She is likely to have permanent hypoparathyroidism due to surgery

Her hypocalcaemia is likely to be transient due to local trauma at the time of


surgery

Your
answer

Her hypocalcaemia is likely to be related to an acute fall in thyroid hormone


concentration after thyroid gland removal and she will recover

Her hypocalcaemia is probably due to a fall in calcitonin after thyroidectomy

Her hypocalcaemia is probably due to coexistent vitamin D deficiency and is


likely to respond to treatment with vitamin D

Up to 10% of patients who undergo subtotal thyroidectomy suffer transient hypocalcaemia.


Thankfully, it becomes permanent hypoparathyroidism in less than 1% of patients. Management
includes calcium chloride infusion (slowly) under ECG monitoring, which should lead to the rapid
resolution of symptoms. Expert thyroid surgeons have now minimised the risk of permanent
hypoparathyroidism.
103.
You review a 32-year-old woman with relapsed Graves disease. TSH is
less than 0.05 mU/l, with a free T4 of 32.5 pmol/l. She has severe bilateral
thyroid eye disease with marked orbital oedema and proptosis. You are
considering radiotherapy as she has failed drug treatment. Which of the
following statements best fits the management of her thyroid eye disease?

Systemic steroids are of no value in managing the eye disease

Orbital irradiation is commonly used to treat thyroid eye disease

Corrective eye muscle surgery should now be considered

Your answer

She should not be given methylcellulose drops as these may worsen


oedema

Her thyroid eye disease may be worsened by radioiodine treatment

Correct
answer

Exacerbation of eye disease is more common after radioiodine treatment (15% vs 3% on drug
therapy alone). Only 510% of these worsening cases threaten sight, but discomfort and
deteriorating cosmetic appearance may cause much patient anxiety. Systemic steroids and oral
non-steroidal anti-inflammatory drugs (NSAIDs) may ease the discomfort and decrease
inflammation in severe symptoms. Orbital irradiation should be reserved for all but the most
severe of cases. Corrective eye muscle surgery should be delayed until thyroid eye disease has
been stable for at least 6 months, but may be of value in improving diplopia.
104.
A 34-year-old woman with a long history of type-1 diabetes,
microalbuminuria and diabetic eye disease presents for review. She has been
amenorrhoeic for 12 months and recent pregnancy tests have been negative.
She has a healthy 1-year-old child but suffered a postpartum haemorrhage
just after his birth. Over the past few months she has also been increasingly
tired and has noticed thinning of her pubic and axillary hair. On examination
she is slim and pale, with small breasts and thin or largely absent pubic and
axillary hair. ACTH, FSH, LH are all low, with TSH just below the normal range.
MRI of the pituitary gland reveals an empty sella. What diagnosis fits best
with this clinical picture?

Prolactinoma

Weight-related pituitary failure

Polyglandular syndrome

Sheehans syndrome

Your answer

Metastatic carcinoma

Sheehans syndrome is well known to occur in women due to postpartum haemorrhage and
hypovolaemic shock, but the risk of it occurring is increased in women with type-1 diabetes who
have microvascular disease, and in patients with sickle-cell anaemia. It is said to occur in 1 in
10,000 deliveries. Initial management includes immediate steroid therapy with later full

endocrine assessment and replacement of pituitary-dependent hormones (eg thyroxine) as


required. Further conception may be difficult and require pulsed gonadotrophin therapy to
restart ovulation. Diabetes insipidus is not usually associated with the syndrome.
105.
A 23-year-old woman is admitted to the Casualty department from her
office, for the third time in the space of 2 months, after having suffered a
syncopal attack. On questioning she admits to feeling very tired over the past
few months and being dizzy on a number of occasions. On examination she
looked slim and tanned, her blood pressure was 110/70 mmHg lying, but
dropped to 85/65 mmHg on standing. ACTH is markedly raised, free thyroxine
is below the lower limit of normal and cortisol is low. Which diagnosis fits best
with the clinical picture?

Hypothyroidism

Primary hypoadrenalism

Your answer

Psychiatric symptoms

Hypovolaemia

HIV
This woman has a marked postural drop, increased pigmentation due to her high ACTH and a
low cortisol, making primary hypoadrenalism the most likely diagnosis. Free thyroxine may also
be low at time of diagnosis, but thyroid hormone replacement should not be started as this may
worsen the adrenal crisis. Instead, steroid replacement therapy should be started, accompanied
by fluid replacement, and thyroid function reassessed at a later stage. Autoimmune destruction
of the adrenal glands is responsible for 80% of cases. Tuberculosis accounts for a further 15%
of cases. There is a female: male predominance of 2:1, and a prevalence of around 5 per
100,000.
106.
A 54-year-old publican is referred by his GP for endocrine assessment.
He is obese with a BMI of 32 and has hypertension, which is poorly controlled
on atenolol, ramipril and bendrofluazide. A recent fasting blood glucose test
has revealed type-2 diabetes. On examination he looks cushingoid with a
blood pressure of 150/95 mmHg. You order a 24-h urinary free cortisol
estimation, which turns out to be just above the normal range. An overnight
dexamethasone suppression test is also unremarkable. Which diagnosis fits
best with this clinical picture?

Cushings disease

Pseudo-Cushings

Simple obesity

Your answer

Essential hypertension

Primary aldosteronism

Obese patients who consume alcohol to chronic excess may acquire a cushingoid appearance.
In this case, this mans occupation as a publican suggests that he may have easy access to
alcohol. His two screening tests for Cushings disease the dexamethasone suppression test
and 24-h urinary free cortisol are normal, which effectively rules out option A. His type-2
diabetes is likely to be related to obesity and his sedentary lifestyle, although diabetes mellitus
could, of course, also be due to alcohol-induced chronic pancreatitis.

Management involves lifestyle measures to promote weight loss, and strict control of his
alcohol intake. Metformin would be the ideal treatment for his diabetes, although this would be
contraindicated in the presence of continued alcohol excess.

107.
You are asked to review a 36-year-old man who has suffered a
myocardial infarction. He is a non-smoker with no past history of note and is
not diabetic. On admission his total cholesterol was 10.2 mmol/l, with
triglycerides just above the normal range, normal HDL and markedly raised
LDL cholesterol. His father died of a myocardial infarction at the age of 43.
What is the most likely cause of his raised cholesterol?

Heterozygous familial hypercholesterolaemia

Your answer

Familial combined hyperlipidaemia

Familial hypertriglyceridaemia

Secondary hyperlipidaemia

Remnant hyperlipidaemia

Heterozygous familial hypercholesterolaemia is an autosomal-dominant monogenic disorder


present in 1 in 500 of the population; the prevalence is increased in French-Canadians, Finns
and South Africans. Homozygous FH occurs in 1 in 10 6 patients and is associated with early
cardiovascular death in childhood. Clinical features may include tendon xanthomas and
xanthelasma. There is a genetic abnormality of the liver LDL receptor. It is said that 50% of men
with this disorder will die by the age of 60 from cardiovascular disease if untreated. High-dose
statin therapy is the standard therapy, and specialist lipidologist advice is recommended for
these patients.
108.
A 28-year-old man presents with acute pancreatitis. He admits to
occasionally drinking wine, but not to excess, and there have been no

symptoms to suggest gall-bladder disease. He suffered a left retinal vein


thrombosis 2 years ago. Triglyceride concentration was estimated at 10
mmol/l, with normal HDL and LDL levels. What is the most likely cause of his
clinical presentation?

Secondary hyperlipidaemia

Familial hypercholesterolaemia

Familial hypertriglyceridaemia

Your answer

Hypolipidaemia

Abetalipoproteinaemia
Triglyceride concentrations above 6 mmol/l carry a significant risk of complications, and this
unfortunate man has suffered both a retinal vein thrombosis and acute pancreatitis. Inherited
lipoprotein lipase or apoprotein C-II deficiency may be responsible, but these defects are rare
and normally present in childhood with eruptive xanthomas, lipaemia retinalis, pancreatitis and
retinal vein thrombosis. So-called familial hypertriglyceridaemia presenting in this way, with
complications in adulthood and no specific identified genetic defect, is more common. Treatment
in this case would involve the avoidance of alcohol and use of a fibrate-type agent.
109.
A 24-year-old man with learning difficulties presents for review. He
complains of a sudden deterioration of vision in his left eye. His past history of
note includes a deep vein thrombosis. On examination he appears tall and slim
and almost marfanoid in appearance. He has a markedly elevated urinary
homocysteine. Which enzyme defect is most likely to be responsible for this
clinical picture?

Methylene tetrahydrofolate reductase

Your answer

Histidase

Homogentisic acid oxidase

Branch-chain ketoacid dehydrogenase

Cystathionine synthetase

Correct answer

This is the presentation of type-1 homocystinuria, where a defect in cystathionine synthetase is


responsible. Patients present with mild to moderate mental handicap, a marfan-like syndrome

and thrombotic episodes. The sudden visual deterioration may be due to a thrombotic episode
or to the lens dislocation associated with this condition. Homocystinuria may also associated
with defects in methylene tetrahydrofolate reductase (type-2 disease), but survivors often have
more severe mental retardation and rarely survive the neonatal period.
110.
A 54-year-old man with long-standing type-2 diabetes presents for
review. He has a history of hypertension, 20 cigarettes per day smoking habit;
recent Hb A1c results have averaged 8.9%. He wore deck shoes without socks
on a recent holiday in Spain, and his wife noticed a large ulcer over the big toe
on his left foot. On examination there is obvious loss of sensation, probing of
the depth of the ulcer elicits no pain. His foot appears warm with a bounding
dorsalis pedis pulse, and there is some toe clawing. What is the likely cause of
his ulcer?

Simple trauma with no underlying pathology

Peripheral vascular disease

Local trauma combined with diabetic neuropathy

Your answer

Vasculitis

Self-neglect

Some 1015% of patients with diabetes suffer foot ulceration at some stage during their lives;
while 50% of all lower limb amputations are performed on patients with diabetes, effective
supervision may prevent a number of these from occurring.

This man has the features of neuropathy, although he certainly has risk factors for peripheral
vascular disease. The neuropathic foot is said to be painless, or to have abnormal neuropathic
pain, to be high arched with toe clawing. It is often warm with bounding pulses and ulceration
tends to occur on the plantar surface. Peripheral ischaemia is said to be associated with rest
pain in a cold and nearly pulseless foot. There is often gravity-dependent reddening of the
foot, which vanishes if the foot is elevated, ulceration tends to be painful and often presents in
the heal area. Of course, it is perfectly possible for the two conditions to coexist, and for a
mixed ischaemic/neuropathic pathology to be the underlying cause.

111.
A 69-year-old woman presented via her GP with episodes of facial
flushing and diarrhoea. An ultrasound scan revealed multiple hepatic lesions,
and a 24-h urine collection revealed an elevated 5-HIAA. Unfortunately she did
not consent to follow-up and next presented 2 years later. Her son noticed a
gradual deterioration in her condition so that she appeared unable to cope at
home. There was apathy, depression and the onset of mild confusion. The
diarrhoea is still present at review, and now she appears to have
photosensitive dermatitis, glossitis and angular stomatitis. What diagnosis fits
best with her clinical picture?

Pellagra

Your answer

Alzheimers disease

Coeliac disease

Riboflavin deficiency

Thiamine deficiency

In the presence of hepatic metastases, diarrhoea, facial flushing and a raised urinary 5-HIAA
(5-hydroxyindoleacetic acid), it is highly likely that this woman has the carcinoid syndrome.
Unfortunately during the 2 years that she has been lost to follow-up, it appears that she has
developed pellagra.

Pellagra is caused by a deficiency of niacin. Niacin is manufactured via an enzymatic pathway


that involves the metabolism of tryptophan. Tryptophan is also the substrate amino acid used
by carcinoid tumours to produce 5-hydroxytryptamine. As the carcinoid tumour mass
increases, more and more of the available tryptophan is consumed, and less is available for
niacin production. Eventually patients may become niacin deficient, suffering the triad of
dermatitis, diarrhoea and dementia, the features of pellagra.

112.
A 45-year-old woman is referred to the endocrine clinic by her GP for
review. She has a body mass index of 35, hypertension and impaired glucose
tolerance. By the time she visits you in clinic she has succeeded in losing 3 kg
in weight. You decide to give her a trial of orlistat and behavioural therapy.
Which of the following best describes the mode of action of orlistat?

Orlistat is a centrally acting appetite suppressant

Orlistat is a pancreatic and gastric lipase inhibitor

Orlistat reduces hepatic glucose production

Orlistat reduces insulin resistance

Orlistat is a b3-agonist

Your answer

Orlistat, a pancreatic lipase inhibitor, blocks the breakdown and hence absorption of dietary fat.
In essence, this means that ingested fat continues its passage through the gut. If patients
taking orlistat do not maintain a low-fat diet then they may suffer distressing oily diarrhoea.
This is why it is essential to combine orlistat therapy with an effective patient support
programme. Sibutramine is a centrally acting drug that acts on serotoninergic and noradrenergic
pathways to encourage earlier satiety and the ingestion of smaller meal portions.
113.
A 42-year-old man with long-standing HIV infection presents for
review. He has been taking antiretroviral therapy for 5 years and has been
relatively free of associated disease. You notice on examination that he
appears to have lost subcutaneous fat on his arms, legs and face, and has
increased deposition of fat around his abdomen. His lipids are also abnormal,
with a raised triglyceride level and low HDL cholesterol. What is the most
likely cause of this clinical picture?

Antiretroviral-related lipodystrophy

Your answer

HIV wasting

HIV-associated malignancy

Likely gastrointestinal pathology

An inherited insulin-resistance syndrome

As the survival of HIV-positive patients improves, more complications of antiretroviral therapy


(ARVs) are becoming apparent. These include a lipodystrophy-type syndrome characterised by
the loss of peripheral and facial subcutaneous fat, but increased abdominal and visceral fat
deposition. There is also an increase in the size of the dorsocervical fat pad buffalo-hump. This
fat redistribution is associated with a picture of abnormalities usually associated with insulin
resistance, such as impaired glucose tolerance, low HDL cholesterol and high triglycerides.
Glitazones may be of value in treating the condition, although some small trials have proved
equivocal.
114.
You are asked by the psychiatrists to review a 42-year-old woman who
has long-standing bipolar disorder for which she takes lithium. During a recent
inpatient stay she appeared to be drinking vast amounts of water and getting
up many times in the night to urinate. Urea and electrolyte testing reveals
elevated sodium and urea concentrations, suggesting possible mild
dehydration. Water deprivation testing reveals a progressively rising serum
osmolality to above 300 mOsm/kg, without increased urine osmolality. Blood
glucose is normal. What is the most likely cause of this clinical picture?

Cranial diabetes insipidus

Your answer

Nephrogenic diabetes insipidus

Correct answer

Psychogenic polydipsia

Diabetes mellitus

Syndrome of inappropriate ADH secretion

This woman has nephrogenic diabetes insipidus (DI) secondary to chronic lithium therapy. There
is usually some recovery after withdrawing lithium therapy but many patients suffer some
degree of permanent nephrogenic DI. Lithium therapy should only be discontinued with
psychiatric advice, but at least now there are possible substitute medications including valproate
and lamotrigine. Other drugs that may cause DI include demeclocycline and glibenclamide.
115.
A 35-year-old woman is referred by her GP because of recurrent
headaches. These tend to come on at times of stress or exercise and appear
almost in a flash. She also complains of intermittent palpitations and
problems with sweating. He has tried her on a course of antidepressants,
which only seemed to make her symptoms worse. Her 24-h urinary
catecholamines are markedly raised. An MRI scan reveals a mass in the right
adrenal medulla. Her blood pressure in clinic is 145/95 mmHg.
What is the best management plan?

Urgent

-blockade

Urgent surgery

Urgent a-blockade, then

-blockade if required, and surgery

Correct answer

Observation

-Blockade followed by urgent surgery

Your answer

This woman has a phaeochromocytoma of the right adrenal medulla. The commonest presenting
features are headache (80%), palpitations (70%), hyperhydrosis (60%) and hypertension
(sustained in 55% and paroxysmal in 45% of patients). Drugs that inhibit catecholamine
reuptake, such as tricyclic antidepressants and cocaine, can exacerbate or unmask the
symptoms of this condition. MRI or MIBG (metaiodobenzyl guanidine) scanning are the definitive
methods of localisation. Management involves a-blockade prior to surgery to avoid a
hypertensive crisis. The 5-year survival rate is 95% for patients with benign
phaeochromocytoma, but this falls to 40% in those with malignant disease.
Phaeochromocytomas are three times more likely to be malignant in women.
116.
A 25-year-old woman presents with a lump on the left-hand side of her
neck in the thyroid region. Thyroid function is normal, and uptake scanning
reveals it to be a cold nodule. Fine-needle aspiration biopsy reveals

architecture suspicious of follicular carcinoma of the thyroid; this is confirmed


on thyroid lobectomy.
Which of the following is the most appropriate management plan?

Oral thyroxine therapy

No further surgery

Your answer

Total thyroidectomy

Total thyroidectomy, radioiodine therapy with oral thyroxine replacement Correct


therapy
answer

Radioiodine therapy

The definitive management for follicular carcinoma of the thyroid without metastases is total
thyroidectomy, followed by radioiodine therapy, with thyroxine replacement to a TSHsuppressive dose. If metastases are present they usually respond to radioiodine therapy, and
this is added. Metastases are subject to haematological spread, with bone being the commonest
metastatic site. The 5-year survival rate approaches 80% for follicular carcinoma. This is much
higher than for anaplastic carcinoma of the thyroid, which usually occurs in the elderly and only
carries a 5-year survival rate of 5%.
117.
A 58-year-old woman presents with fever, marked anxiety and
agitation, palpitations, marked muscle weakness and diarrhoea. On
examination she has goitre, and is in fast atrial fibrillation with a ventricular
rate of 135 beats per minute. Urine testing reveals evidence of protein, white
cells and red blood cells. Thyroid function testing reveals a TSH of < 0.05
mU/l. Which of the following diagnoses best fits this clinical picture?

Urinary tract infection

Paroxysmal atrial fibrillation

Hypothyroidism

Phaeochromocytoma

Thyroid storm

Your answer

Thyroid storm is an unusual presentation of thyrotoxicosis, precipitated by acute stress in a


previously undiagnosed patient. Causes include infection (as in this case), myocardial
infarction or diabetic ketoacidosis. Inadequate adherence to therapy in an already diagnosed
patient may also be responsible. Symptoms include fever, anxiety and agitation, sweating,
marked proximal muscle weakness, tachycardias, diarrhoea and sometimes heart failure,
particularly in the elderly.

Acute general management includes carbimazole therapy, b -blockade, digitalisation of atrial


fibrillation and occasionally corticosteroids, which may inhibit thyroid hormone release. Lugols
iodine, used in conjunction with carbimazole therapy may bring a more rapid resolution.

118.
A 54-year-old thyrotoxic man has been treated with radioiodine. What
is the best advice for the patient during the post-radioiodine period?

Antithyroid drugs should never be taken after radioiodine treatment

He should not have close contact with children under the age of 11 years for Your
about 2 weeks after treatment
answer

There is no need to monitor his TSH level

He will never need further doses of radioiodine

He may have close contact with children within 34 days


Antithyroid drugs may be recommenced after radioiodine administration, but treatment should
be withdrawn gradually and guided by 68 weekly TFTs. Early post-radioiodine hypothyroidism
may be transient. The patients TSH level should be monitored every 6 months after radioiodine
therapy to determine late hypothyroidism. Patients should not have close contact with children
under the age of 11 years for about 2 weeks after treatment; this is to prevent exposing
children to radioactivity. However, there is no overall excess risk of cancer.
119.
A 61-year-old patient with a history of recent thyrotoxicosis underwent
major surgery a week ago. He now presents with altered mental status,
tachycardia, high-grade fever, vomiting and cardiac failure. A diagnosis of
thyroid storm (crisis) is made. What is the most important next step in
management?

Transfer the patient to ITU

Avoid chlorpromazine in the treatment of agitation

Avoid carbimazole treatment

Your answer

Potassium iodide should be given immediately before propylthiouracil

Peritoneal dialysis should be started immediately

Any acute stressful precipitating factor such as surgery can result in a thyroid storm or crisis.
This can also occur in patients with acute infections, postpartum and when antithyroid drugs
are being withdrawn. Thyroid crisis is associated with a significant mortality rate (3050%)
and is best managed in an intensive care unit where close attention can be paid to
cardiorespiratory status, fluid balance and cooling.

Chlorpromazine can be used to treat agitation and, because of its effect in inhibiting central
thermoregulation, it may be useful in treating the hyperpyrexia. Specific treatment includes
propylthiouracil (PTU), potassium iodide (after starting PTU), -blockers and glucocorticoids.
There is no clinical data comparing PTU and carbimazole in this situation. Plasmapheresis and
peritoneal dialysis may be effective in cases resistant to pharmacological measures.

120.
A 25-year-old woman develops hyperthyroidism 6 weeks after delivery.
On examination she has painless, firm enlargement of the thyroid gland.
Which of the following statements is most correct with regards to the probable
diagnosis?

The condition is more likely in those in whom thyroid peroxidase (TPO)


antibodies were positive prior to delivery

Correct
answer

When followed up, most patients have lifelong hypothyroidism

It is less common in patients with a history of Type-1 diabetes

Around 20% of women have some degree of thyroid dysfunction postpartum

Your
answer

If it is associated with intense fibrosis of the thyroid gland

Postpartum thyroiditis is thyroid dysfunction occurring within the first 6 months postpartum.
Prevalence ranges from 5 to 7%. It develops in 3052% of women who have positive TPO
antibodies. Most patients have a complete remission but some may progress to permanent
hypothyroidism. It is twice as common in patients with type-1 DM.

121.
A 27-year-old woman on bromocriptine for microprolactinoma becomes
pregnant. What is the most appropriate management advice?

Continue bromocriptine at the same dose

Your answer

Visual field testing to be done every fortnight

Continuing bromocriptine would significantly increase congenital


abnormalities

Stop bromocriptine as soon as pregnancy is confirmed

Correct
answer

Needs cerebral magnetic resonance imaging (MRI) at least twice during


pregnancy

Whilst bromocriptine is an effective treatment for microprolactinoma in terms of suppression of


prolactin secretion, it may interfere with the natural rise in prolactin seen in pregnancy.
Bromocriptine should therefore be stopped as soon as pregnancy is confirmed. Visual field
testing and MRI are indicated only in the occasional patient who becomes symptomatic. Data on
children whose mothers received bromocriptine throughout pregnancy indicate that the
incidence of congenital abnormalities is negligible; hence the actual risk of possible harm is low.
122.
A 41-year-old heavy smoker presents with a serum sodium level of 112
mmol/l. A diagnosis of SIADH is confirmed. What is the most appropriate
initial management of his fluid balance?

Immediate normal saline infusion

Fluid restriction

Your answer

Desmopressin

Glucocorticoids

Perform fluid deprivation test


In patients with the syndrome of inappropriate ADH (SIADH) secretion, it is important to restrict
fluids to 5001000 ml/24 hours. The underlying cause must also be detected and treated. If the
problem is not temporary and long-term fluid restriction is likely to be difficult, then
demeclocycline may be effective by inducing partial nephrogenic diabetes insipidus. In an

emergency, saline infusion may be required; however, great care is required as rapid
overcorrection of hyponatraemia may cause central pontine myelinolysis.
123.
You are called to the psychiatric unit to see an 18-year-old woman with
anorexia nervosa. What would you expect to see in the results of her
biochemical investigations?

Raised LH, FSH

Elevated circulating cortisol

Correct answer

Low resting growth hormone levels

Your answer

Increased GnRH

Normal oestrogen levels


The typical patient with anorexia nervosa is a woman aged < 25 years with weight loss,
amenorrhea and behavioural changes. There is a long-term risk of severe osteoporosis.
Endocrine abnormalities include GnRH deficiency, low LH and FSH, low oestrogen in women,
raised circulating cortisol, low to normal thyroxine, reduced T3, normal TSH and increased
resting GH levels.

124.
A 72-year-old woman recently diagnosed as having
hyperparathyroidism has a serum calcium concentration of 3.2 mmol/l. What
is the most appropriate treatment?

Surgery

Your answer

Long-term observation with regular blood tests

Long-term bisphosphonates

Hormone replacement therapy

Do further tests to see if there is end-organ damage

In primary hyperparathyroidism, a patient with a markedly elevated serum calcium > 3 mmol/l
should be referred for surgery, unless this is contraindicated for other reasons. Other indications
for surgery include impaired renal function, renal stones, nephrocalcinosis, reduced bone

mineral density (BMD), substantially elevated urinary calcium excretion of >10 mmol/24 hours.
Medical management is only indicated if the patient is unsuitable for surgery.
125.
What is the commonest cause of death in patients with von Hippel
Lindau disease?

Cerebellar haemangioblastoma

Your answer

Renal carcinoma

Correct answer

Retinal tumours

Myocardial infarction

Phaeochromocytoma
VHL disease is an autosomal-dominant condition, with the VHL gene being located on
chromosome 3. Estimated prevalence is 1/39,000. Retinal angiomatosis is the initial
presentation in 40% patients. Cerebellar haemangioblastoma is a common initial presentation.
Renal carcinoma is the commonest cause of death, with phaeochromocytoma occurring in 40%
of patients with VHL.
126.
A 30-year-old man arrives in A&E in an unconscious state. His initial
blood glucose reading by monitor is 2.1 mmol/l. What is the most important
immediate management of this patient?

Oral glucose

Mannitol

50% glucose into a large vein after first taking a blood sample

Your answer

10 mg glucagon IM

Check his serum ethanol concentration


Hypoglycaemia is defined as a plasma glucose concentration < 2.5 mmol/l and associated with
symptoms of neuroglycopenia. Large-bore venous access should be achieved and blood samples
for serum glucose, liver function, ethanol, cortisol, insulin, C-peptide, proinsulin and
sulphonylurea levels should be sent for testing. If the patients Glasgow Coma Score (GCS) is <
13, then 2550 ml 50% glucose should be given intravenously. Glucagon (1 mg im) may be
administered if no intravenous access can be obtained.

127.
A 28-year-old man presents with erectile dysfunction. What is the
commonest cause in this age group?

Alcohol

b-Blockers

Diabetes mellitus

Psychological factors

Your answer

Testicular tumour

Erectile dysfunction is present in approximately 10% of all men and >50% of men over the age
of 70 years. Common causes are psychological factors (20%), drugs (25%) and endocrine
causes. Baseline investigations include serum testosterone, prolactin, fasting glucose, LH and
FSH, thyroid function tests, liver and renal function, lipids and ferritin.
128.
Which of the following is the most likely long-term consequence of the
menopause?

Decreased thrombotic tendency

Less likelihood of developing ischaemic heart disease (IHD)

Increased possibility of developing Alzheimers dementia

Correct answer

Increased bone mineral density (BMD)

Increased insulin sensitivity

Your answer

During the perimenopausal period there is an accelerated loss of BMD, rendering


postmenopausal women more susceptible to osteoporotic fractures. They are also more likely to
develop IHD, decreased insulin sensitivity and increased thrombotic tendency. Women are 23
times more likely to develop Alzheimers disease than men. It is suggested that oestrogen
deficiency may play a role in the development of dementia.
129.
Which of the following in a 64-year-old man with diabetes mellitus
warrants urgent ophthalmology referral?

Background diabetic retinopathy

Vitreous haemorrhage

Your answer

Cataracts

Drusen

Non-proliferative changes in the periphery


Immediate referral is needed for patients with proliferative retinopathy (as it carries a 40% risk
of blindness if untreated and laser treatment reduces this), rubeosis iridis, vitreous
haemorrhage, advanced retinopathy with fibrous tissue, retinal detachment or maculopathy.
130.
In which of the following coexisting medical conditions would
prescription of the oral contraceptive pill (OCP) be acceptable?

Severe migraine

History of venous thrombosis

Liver disease

Pulmonary hypertension

Your answer

Hypothyroidism

Correct answer

Absolute contraindications for OCP includes a history of heart disease, pulmonary hypertension,
history of arterial or venous thrombosis, history of cerebrovascular disease, liver disease, severe
migraine, breast or genital tract cancer and age over 35 years. Hypothyroidism is not a
contraindication to taking the OCP.
131.
A 28-year-old pregnant woman is detected to have positive thyroid
antibodies and is euthyroid. Which of the following potential consequences
should the obstetrician warn of during early pregnancy?

Higher risk of spontaneous abortions

Correct answer

Increased risk of neonatal hypothyroidism

Your answer

Increased risk of pregnancy-induced hypertension (PIH)

Increased fetal size

Increased risk of thyroid cancer in the mother

A pregnant mother with positive thyroid antibodies, but who is euthyroid, has a higher risk of
spontaneous abortions. There is no risk of neonatal hypothyroidism and the risk of PIH is not
increased. The occasional mother will develop hypothyroidism at the end of pregnancy, so the
TSH level should be checked between 28 and 32 weeks gestation, then at 3 months
postpartum.
132.
A 26-year-old woman presents with symptoms of flushing, diarrhoea
and wheezing. After examination a clinical diagnosis of carcinoid syndrome is
suspected. Which of the following tests would be the most sensitive marker
for carcinoid syndrome?

Urinary 5-HIAA

Your answer

Alkaline phosphatase

Echocardiography

Plasma chromogranin A

Correct answer

Plasma gut-hormone profile

The most sensitive marker for carcinoid syndrome is plasma chromogranin A, which has been
found in 100% of patients. Although the specificity is lower than for 5-HIAA (5hydroxyindoleacetic acid), most tumours with neuroendocrine differentiation are associated with
increased levels of plasma chromogranin A. This test is not widely available for clinical use.
Urinary 5-HIAA has a sensitivity of about 70% and a specificity of 100% in patients with
carcinoid syndrome. Alkaline phosphatase may remain normal despite liver involvement and a
plasma gut-hormone profile may be useful.
133.
What is the most appropriate investigation to confirm a diagnosis of
acromegaly biochemically after initial screening?

Increased IGF-1

Your answer

Random growth hormone (GH) assay

Insulin tolerance test

Thyroid function test

Oral glucose tolerance test (OGGT) with growth hormone assay

Correct answer

In acromegaly, there is a failure to suppress GH to < 2 mU/l in response to a 75-g oral glucose
load. In contrast, the normal response is GH suppression to undetectable levels. Random GH is
not useful in the diagnosis of acromegaly because, although normal subjects have undetectable
growth hormone levels throughout the day, there are pulses of GH which are impossible to
differentiate from the levels seen in acromegaly. Following a TRH (thyrotrophin-releasing
hormone) test, 80% of patients with acromegaly show increased levels of GH. IGF-1 is useful for
initial screening for acromegaly, as it provides an integrated measure of growth hormone levels
over time.
134.
An 18-week pregnant woman presents to hospital with failure to gain
weight, hyperemesis and persistent tachycardia. She is found to be thyrotoxic
with a suppressed TSH of 0.05.
Which of the following is the best management step?

Radioiodine

Immediate surgical referral

Block-and-replace regime

Propylthiouracil

Your answer

Observe and wait for normalisation


The aim of treatment of thyrotoxicosis in pregnancy is the alleviation of symptoms and
normalisation of tests in the shortest time. Both propylthiouracil and carbimazole are effective in
controlling disease in pregnancy. The use of radioiodine is contraindicated in pregnancy. Surgery
is rarely performed during pregnancy and is reserved for patients who do not respond to
antithyroid drugs. A block-and-replace regime should not be used as this will result in fetal
hypothyroidism. This is a different scenario to that of hyperemesis and suppressed TSH, which is
seen in the first trimester and usually resolves.

135.
A 34-year-old woman presents with 4-month history of anorexia and
weight loss, excess pigmentation and dizziness on standing. Initial
investigations reveal hyponatraemia and hyperkalaemia. Which of the
following tests will be most useful to confirm the diagnosis?

ESR

Thyroid function tests

Serum urea

Short Synacthen test

Your answer

Serum calcium

Hyponatraemia is present in 90% and hyperkalemia in 65% of patients with primary adrenal
insufficiency. Although increased urea, increased ESR and raised TSH levels and mild
hypercalcaemia are seen in Addisons disease, these are not particularly useful in confirming the
diagnosis. However, failure to respond following a short Synacthen test suggests adrenal failure.
136.
A 72-year-old man with a long history of COPD was admitted with
pneumonia. Prior to admission he had become increasingly confused. On
examination he was drowsy, had a BP of 142/75 mmHg and was clinically
euvolaemic. Chest auscultation was consistent with pneumonia. Blood
biochemistry investigations revealed Na 121 mmol/l, K 3.9 mmol/l, urea 2.4
mmol/l, creatinine 64 mol/l and glucose 4.2 mmol/l. Plasma osmolarity was
261 mOsmol/kg, and thyroid function testing was normal. Urine testing
revealed an osmolality of 560 mOsmol/kg and a sodium concentration of 55
mmol/l.
What was the most likely cause of his hyponatraemia?

Addisons disease

Renal failure

Cardiac failure

Cirrhosis

Syndrome of inappropriate ADH secretion (SIADH)

Your answer

This is a not uncommon consequence of pneumonia, particularly in patients of this age group.
Small-cell bronchial carcinoma is another cause of SIADH. Diagnosis is made on an
inappropriately high urine osmolality compared to plasma osmolality, urinary sodium
concentration above 30 mmol/l, euvolaemia and normal thyroid and adrenal function. The
normal potassium level here makes Addisons disease unlikely. A number of drugs can cause of
SIADH, including chlorpropamide, phenothiazines and carbamazepine. Both fluid restriction and
demeclocycline precipitate renal diabetes insipidus, which may be of assistance in the treatment
of SIADH.
137.
A 45-year-old woman is due to undergo a hysterectomy. She has a
previous history of hypertension, headaches and panic attacks over the past
few years. She currently takes ramipril for her blood pressure. The
preoperative examination revealed a blood pressure of 150/85 mmHg, normal
renal function and calcium concentration, an ECG showed mild LVH.
Unfortunately during the operation, as the surgeon attempts to mobilise her
uterus, her blood pressure rises to 210/110 mmHg, her pulse to 130 bpm and
she suffers an acute myocardial infarction. What is the most likely cause of her
intraoperative hypertension and myocardial infarction?

Essential hypertension

Occult coronary artery disease

Undiagnosed phaeochromocytoma

Your answer

MEN-1

Renal artery stenosis


Given her history, she is likely to have been suffering paroxysms of catecholamine release.
Mobilisation of the uterus is likely to have precipitated an acute release of large amounts of
catecholamines, causing her catastrophic rise in blood pressure and intraoperative myocardial
infarction. Standard therapy for phaeochromocytoma is aggressive -blockade prior to surgery,
then surgical excision. The 5-year prognosis for benign tumours approaches 95%, but this falls
to around 40% in patients with malignant disease.
138.
A marfanoid-looking, 21-year-old man is referred by his GP for review.
He has been suffering from acute headaches and panic attacks and is unable
to continue his course work at university. On examination his blood pressure
is 148/98 mmHg and you notice a number of neuromas around his lips. His
24-h urinary catecholamine levels are raised.
What diagnosis fits best with this clinical picture?

Multiple endocrine neoplasia (MEN)-1

Multiple endocrine neoplasia (MEN)-2a

Neurofibromatosis

Multiple endocrine neoplasia (MEN)-2b

Your answer

Carcinoid syndrome
MEN-2b is characterised by a marfanoid habitus, visceral and intestinal ganglioneuromas (which
may occur around the lips and tongue), adrenal tumours, and medullary thyroid carcinoma,
parathyroid hyperplasia occurs much more rarely than in MEN-2a. Adrenal tumours leading to
Cushings syndrome or phaeochromocytomas (70% bilateral) may occur. Screening is
recommended for MEN-2: total thyroidectomy in childhood is often recommended for those with
a known gene defect. Screening for medullary thyroid carcinoma is via the pentagastrin test.
The best screen for phaeochromocytoma is 24-h urinary catecholamine estimation.
139.
A 45-year-old man presents for review at the type-2 diabetes clinic. He
is on maximal metformin and his Hb A1c is still 7.8%. You elect to add in a
peroxisome proliferator-activated receptor (PPAR)-gamma agonist,
pioglitazone. Which of the following best describes the mode of action of
PPAR- agonists?

They act at the PPAR- receptor site, promoting binding as a heterodimer


with the retinoid X-receptor to DNA

Your
answer

They bind to a promoter region of DNA as a heterodimer with the retinoid Areceptor

They act by reducing peripheral insulin sensitivity

They act by stimulating insulin output

They act via receptors at the cell surface


PPAR- agonists act by binding to the PPAR-gamma receptor, which binds to a promoter region
of DNA in tandem with the retinoid X-receptor. This then leads to the upregulation of a number
of enzymes concerned with lipid metabolism, bringing about a fall in free fatty acids. The fall in
free fatty acids promotes a reduction in hepatic insulin resistance, a rise in adiponectin and
improved peripheral insulin sensitivity. Retinoid X-receptors are also now under investigation for
their possible action in improving insulin sensitivity. Currently, there are two PPAR- agonists on
the market, rosiglitazone and pioglitazone, both with promising data for long-term glycaemic
control in type-2 diabetes.
140.
A 52-year-old woman is referred by the A&E department after
attending with a Colles fracture, the second in the last 3 years. She
underwent a total hysterectomy at the age of 38 for carcinoma. Bone
densitometry confirms osteoporosis. The calcium and parathyroid hormone
assays are normal. Which of the following is the most likely concerning her
underlying pathophysiology?

She has suffered predominantly cortical bone loss

She has probably suffered an equal mix of cortical and trabecular bone
loss

She has probably suffered predominantly trabecular bone loss

Your answer

Underlying hyperparathyroidism will have contributed

Underlying hypoparathyroidism will have contributed

Hypo-oestrogenisation, as in this case, is usually characterised by predominantly trabecular


bone loss. Involutional bone loss of old age is characterised by a mixed picture of trabecular and
cortical bone loss. In the presence of normal calcium and parathyroid hormone levels,
parathyroid disease is somewhat unlikely. Hormone replacement therapy was formerly the
mainstay of treatment, but this has fallen out of favour due to increased thromboembolic
disease and the risk of breast carcinoma. Bisphosphonates are now the favoured treatment
modality in this group of patients.
141.
A 32-year-old aromatherapist is referred to the endocrine clinic for
review. She has been suffering intermittent tachycardias and panic attacks.
There is no significant past medical history. She takes a number of vitamin and
mineral supplements, including kelp. On examination there are no eye signs
and no goitre. TSH is < 0.05 mU/l and thyroid antibodies are negative. What is
the most likely cause of her thyrotoxicosis?

Graves disease

Toxic multinodular goitre

Solitary toxic nodule

Excess iodine ingestion

Your answer

Excess TSH secretion

Kelp is a very rich source of iodine. In patients with pre-existing thyroid hyperplasia or
adenoma, the ingestion of large amounts of iodine may precipitate thyrotoxicosis. Treatment is
withdrawal of the kelp with monitoring of thyroid function. Thyrotoxicosis affects 2% of women
and 0.2% of men in their lifetimes. Graves disease is responsible for around 80% of cases of
thyrotoxicosis; toxic multinodular goitre is the next commonest cause. Patients with Graves
disease may be offered radioiodine, if suitable, or a period of medical therapy with antithyroid

drugs and thyroxine replacement. Patients who then fail on medical therapy are offered
radiotherapy or surgery.
142.
A 17-year-old student presents with intermittent weakness and
feelings of tiredness. Her GP requested some blood tests and found her to
have a potassium level of 2.8 mmol/l and bicarbonate of 32 mmol/l. She is
normotensive. You arrange a renin and aldosterone level, and both are
elevated. Urinary calcium excretion is elevated. Urinary diuretic screen is
negative.
Which diagnosis fits best with this clinical picture?

Diuretic abuse

Gitelmans syndrome

Liddles syndrome

Bartters syndrome

Your answer

Conns syndrome

This syndrome is characterised by increased urinary calcium excretion, hypokalaemia, metabolic


alkalosis and raised renin and angiotensin levels. Age at onset may vary depending on the
severity of the causative mutation, whilst most patients present in childhood, some do present
later. This is not Conns syndrome because the patient is normotensive and both her renin and
angiotensin levels are raised. Bartters syndrome is due to a mutation in the Na +K+2Cl cotransporter, the ATP-regulated potassium channel, or the kidney-specific basolateral chloride
channel. This impairs sodium and chloride reabsorption, and causes an increase in the
production of renal prostaglandin E2. Hyperplasia of the juxtaglomerular apparatus is seen on
renal biopsy. Gitelmans syndrome is due to a different mutation, and produces a clinical picture
similar to that seen with chronic thiazide administration. Liddles syndrome is characterised by
potassium wasting and hypertension.
143.
A 28-year-old man presents with hypertension that his GP is finding
difficult to manage. There are a number of metabolic abnormalities and he is
concerned about the possibility of Conns syndrome. The blood picture is one
of metabolic acidosis, hyperkalaemia and low renin and aldosterone levels.
What diagnosis fits with this clinical picture?

Gordons syndrome

Bartters syndrome

Addisons disease

Your answer

Conns syndrome

Gitelmans syndrome

Gordons syndrome presents as almost the mirror image of the metabolic abnormalities seen in
Bartters syndrome, and is caused primarily by renal sodium retention and volume expansion.
Addisons disease is, of course, associated with hypotension and hyperkalaemia. Conns
syndrome is primary hyperaldosteronism and is associated with hypokalaemia. In general, the
emergency treatment of hyperkalaemia is well documented, with acute administration of
calcium to reduce the risk of arrhythmias, insulin to drive potassium into cells and the use of
ion-exchange resins to increase the excretion of potassium via the gastrointestinal tract.
144.
A 35-year-old woman, with a strong family history of breast cancer,
visits you because she is keen to start on tamoxifen for breast cancer
prophylaxis. Which of the following statements best describes the mode of
action of tamoxifen?

It is a progesterone-receptor agonist

It is a progesterone-receptor antagonist

It is an oestrogen-receptor agonist

It is an oestrogen-receptor antagonist

It is a mixed oestrogen-receptor antagonist and partial agonist

Your answer

The use of tamoxifen after breast cancer is associated with a 25% risk reduction in metastatic
disease, and is also associated with a reduction in the risk of primary breast cancer in highrisk women. It is a mixed antagonist and partial agonist at the oestrogen receptor. Its partial
agonist action is probably reflected in the increased risk of endometrial carcinoma associated
with chronic tamoxifen use. Tamoxifen has been considered the treatment of choice, although
positive evidence for other hormone modulators is accumulating.

Additional modern selective oestrogen-receptor modifiers now exist, one example being
raloxifene which is currently used for the treatment of osteoporosis.

145.
You are asked to urgently review a 58-year-old woman who presents
with a slowly enlarging hard mass in the anterior neck. Thyroid ultrasound
reveals infiltration, and biopsy does reveal dense infiltration of the gland. Free
T4 is low and her TSH is markedly raised, consistent with hypothyroidism.
Thyroid autoantibodies are negative. Which of the following is the most likely
diagnosis given this clinical picture?

Riedels thyroiditis

Your answer

Thyroid carcinoma

Hashimotos thyroiditis

Graves disease

Toxic multinodular goitre


By the nature of its presentation, Riedels thyroiditis is often confused with thyroid carcinoma. It
is characterised by marked fibrous infiltration of the thyroid gland, the aetiology of this being as
yet unidentified. Treatment is with thyroxine replacement. Hashimotos thyroiditis is
autoimmune in aetiology and is characterised by lymphocytic infiltration and the presence of
antimicrosomal antibodies. Graves disease is associated with thyroid-stimulating autoantibodies
and hyperthyroidism. Thyroid ultrasound reveals infiltration but no discrete mass is identified,
and biopsy does reveal dense infiltration of the gland.
146.
You are referred a 15-year-old girl from a family of travellers who has
never previously attended medical care. Her mother is concerned that she has
short stature and is still to commence her periods. On examination she is
clearly below the 50th centile for height. She appears to have excess skin
around her neck, and has poor development of secondary sexual
characteristics. There is a murmur suggestive of aortic stenosis. Chromosome
analysis reveals a 45 X: 0 picture. What diagnosis fits best with this clinical
picture?

Noonans syndrome

Turners syndrome

Your answer

Testicular feminisation

Congenital adrenal hyperplasia

Congenital lymphoedema

Turners syndrome occurs in between 1 in 2500 and 1 in 5000 live births. Rarely, girls with
Turners syndrome may still present late if they have limited contact with medical services, as
in this case. In older children, symptoms include slow linear growth, short stature, delayed
development of secondary sexual characteristics and absence of menses. There may also be
dyspraxia, poor spatial awareness or mild intellectual impairment. Associated cardiovascular

defects may include aortic stenosis, bicuspid aortic valve or coarctation of the aorta. Patients
with Turners syndrome also have an increased risk of hypothyroidism.

Noonans syndrome presents with a similar phenotypic picture to Turners syndrome but
without the absent X chromosome.

147.
A 74-year-old man who is maintained on metformin for type-2 diabetes
presents to the emergency department acutely unwell. He is shocked, drowsy
and confused. Blood testing reveals a metabolic acidosis with an anion gap of
24 mmol/l. Ketones are not significantly elevated and random blood glucose
was 8.7 mmol/l.
What is the mainstay of treatment for this condition?

Intravenous insulin therapy

8.2% sodium bicarbonate

1.26% sodium bicarbonate iv

4.1% sodium bicarbonate iv and rehydration

Rehydration

Your answer

The patient has lactic acidosis and requires close monitoring and should be admitted to an HDU
or ITU ward. The lactate is the cause of the raised anion gap. The mainstay of treatment is
rehydration. Previous commentaries recommended sodium bicarbonate, although no
randomised clinical trials exist to support its use and it may be deleterious. Despite modern fluid
management and aggressive treatment of these patients, mortality still approaches 50%. It is
important to monitor creatinine in patients on metformin as a raised serum creatinine level
significantly increases the risk of lactic acidosis. Many physicians reduce or discontinue
metformin when the serum creatinine level is above 130 mol/l in women or 150 mmol/lin men,
although the serum creatinine level should always be considered in the context of overall muscle
mass.
148.
A 54-year-old type-2 diabetic man presents for review. Which of the
following laboratory test results would be most significantly associated with
an increased incidence of cardiovascular disease in his case?

Raised proinsulin levels

Decreased proinsulin levels

Your answer

Normal or decreased LDL cholesterol

Increased HDL cholesterol levels

Increased triglyceride level to 10% above the normal range

Raised levels of proinsulin were shown in the Caerphilly Cohort Study to be independently
associated with an increased incidence of cardiovascular disease.
Intervention trials with proinsulin were also discontinued after a possible CV event signal was
seen. The reason behind this association has not yet however been clearly elucidated.

Raised LDL levels are an independent cardiovascular risk factor and treatment with statins has
been proven to reduce cardiovascular risk.

Increased HDL cholesterol levels are associated with reverse cholesterol transport and
increased processing of cholesterol by the liver, and are therefore protective.

Increased triglyceride levels are associated with increased cardiovascular risk, but their
importance as a cardiovascular risk factor has yet to be proven categorically via an
intervention study.

Insulin resistance, a root cause of type-2 diabetes, is also independently associated with an
increased cardiovascular risk. Whether it is a risk marker or a risk factor in its own right is
open to debate. Results of glitazone intervention to reduce insulin resistance in Type 2
diabetes have been mixed at best, with the Proactive study demonstrating perhaps a small
benefit in reducing events, and the RECORD study demonstrating at best a neutral effect.

149.
You are reviewing a 52-year-old woman who has a history of type-1
diabetes, autoimmune thyroid disease and coeliac disease. Her daughter has
been researching autoimmune disease on the Internet and has been learning
about the association between certain HLA types and disease. Which of the
following HLA subtypes is most strongly associated with autoimmune thyroid
disease or type-1 diabetes?

HLA-B47

HLA-B27

HLA-DR3

Your answer

HLA-A28

HLA-DR7
HLA-DR3 is associated with diabetes mellitus, autoimmune hepatitis, dermatitis herpetiformis,
Graves disease, membranous glomerulonephritis, myasthenia gravis, Addisons disease,
Sjgrens syndrome and systemic lupus erythematosus (SLE). HLA-B47 is associated with
congenital adrenal hyperplasia, -B27 with seronegative arthropathies, -A28 with schizophrenia
and -DR7 with minimal-change disease (nephrotic syndrome).
150.
An obese 48-year-old woman visits the endocrine clinic. Screening for
endocrine disease, including diabetes mellitus, hypothyroidism and Cushings
disease, is negative. She asks for dietary advice, particularly about her fat
intake. Which of the following is the best advice to give her concerning her fat
intake?

Total fat intake should be restricted to less than 40% of total dietary
energy

Saturated fats should provide no more than 5% of dietary energy

Monounsaturated fats should provide around 6% of dietary energy

Polyunsaturated fats should provide around 3% of dietary energy

Total fat intake should be restricted to less than 30% of total dietary
energy

Your answer

The current recommendations in the UK for fat intake are that total fat intake should be
restricted to less than 30% of dietary energy: that monounsaturated fats should provide around
12%, polyunsaturated fats around 6% and that saturated fats should provide no more than 10%
of dietary energy. Increased dietary fat intake has an epidemiological association with
cardiovascular disease, cancers, obesity and type-2 diabetes. Essential fatty acid deficiency may
occasionally occur in patients on long-term parenteral nutrition who are given a mix of protein
and glucose. Alopecia, thrombocytopenia, anaemia and dermatitis may occur.
151.
You are asked by the respiratory physicians to see a 74-year-old man
who has been admitted from A&E with an abnormal chest X-ray revealing a
right hilar mass. There is a history of 40 years of cigarette smoking. He is
noted to have a markedly raised corrected calcium level of 3.25 mmol/l. Apart
from a mildly raised urea level due to dehydration, his other renal function
testing is normal. He also has a normochromic normocytic anaemia with a
haemoglobin of 10.0 mg/dl.
What is likely to be the underlying cause of his hypercalcaemia?

Primary hyperparathyroidism

Secondary hyperparathyroidism

Tertiary hyperparathyroidism

Pseudo-hyperparathyroidism

Raised parathyroid hormone-related protein

Your answer

Parathyroid hormone-related protein (PTH-rp) is a 144 amino acid polypeptide, the initial
sequence of which shows some homology with the biologically active part of PTH. This is being
produced by his bronchial carcinoma. Common primary tumours which may be associated with
hypercalcaemia include bronchial, breast, renal, prostate carcinomas, myeloma and lymphoma.
Treatment involves rehydration and intravenous bisphosphonate therapy. Patients are often
trialled on prednisolone, but this is rarely successful.
152.
A 19-year-old student nurse was admitted after her third collapse in
recent months. She was noted to have a blood sugar of 0.9 mmol/l on finger
-prick testing and responded well to intravenous glucose therapy. Venous
blood taken at the same time as obtaining venous access showed a markedly
raised insulin level, but her C-peptide levels were normal. What diagnosis fits
best with this clinical picture?

Insulinoma

Glucagonoma

Occult administration of sulphonylureas

Self-administration of a short-acting insulin

Your answer

Type-1 diabetes
This woman has hyperinsulinaemia and hypoglycaemia, but her C-peptide levels are normal.
This pattern is strongly suggestive of the fact that she is self-administering insulin. On further
questioning in this case it was determined that her father has type-1 diabetes and she had been
using his Actrapid to induce hypoglycaemia during times of stress at work. If both the insulin
and C-peptide levels were raised, then it would be worth screening her urine for a sulphonylurea
assay. Only after this had been excluded would it be worth pursuing investigation for
insulinoma.
153.
A 35-year-old woman visits you in the paediatric diabetes clinic with
her 2-year-old son who has recently developed type-1 diabetes. He has an
identical twin brother and she is concerned about his risk of developing
diabetes. What advice would you give regarding his future risk?

He has a 100% future risk of developing type-1 diabetes

A trial of low-dose insulin in the unaffected child will reduce his future risk
of diabetes

GAD antibodies in the unaffected child are not predictive of the risk of
diabetes

He has a 3050% future risk of developing type-1 diabetes

Your
answer

IA-2 antibodies in the unaffected child are not predictive of the risk of
diabetes

Twin studies have suggested that the future risk of diabetes in the unaffected child is 3050%.
The presence of glutamic acid decarboxylase (GAD), islet-cell or IA-2 antibodies in the
unaffected child increases the likelihood that the child will go on to develop type-1 diabetes.
Children of type-1 diabetic patients have a slightly increased risk of developing the disease (3
6% risk where the father is diabetic, 23% where the mother is diabetic). Twin studies in type-2
diabetes have suggested that if one identical twin develops the disease, then the risk in the
unaffected twin rises to 50% or greater. Maturity-onset diabetes of the young (MODY) is a rare
variant of type-2 diabetes with a very strong familial pattern of inheritance.
154.
You are reviewing a 45-year-old man who has type-2 diabetes. He
works a varying shift pattern as a taxi driver and has not tolerated metformin
therapy due to gastrointestinal side-effects. You decide that the postprandial
glucose regulator nateglinide is the most appropriate therapy choice. Which of
the following best describes the mode of action of nateglinide?

It acts by reducing hepatic glucose output

It acts by reducing peripheral insulin resistance

It acts by closure of

-cell calcium channels

It acts by closure of the

-cell KATP channel

Your answer

It prevents the gastrointestinal absorption of sugars

It acts by closure of the -cell K-ATP channel promoting release of insulin. It is short acting and
taken to coincide with meals. Nateglinide is more expensive than sulphonylureas but offers

advantages for shift workers and patients who may fast for a period of time as doses can be
skipped when meals are missed. In these patient groups there may be less incidence of
hyperglycaemia. It appears that the shorter action of duration appears to result in less weight
gain compared to traditional sulphonylureas, and in theory these agents should result in less
strain on the -cell.
155.
A 74-year-old man is admitted to A&E in an acute confusional state. His
serum sodium concentration is 105 mmol/l and because he has heavily
nicotine-stained fingers, SIAD is suspected.
Which of the following biochemical findings best supports this diagnosis?

Random plasma cortisol concentration of 548 nmol/l

Random urine osmolality of 380 mmol/kg

Correct answer

Random urine sodium concentration of 72 mmol/l

Serum osmolality of 230 mmol/kg

Your answer

Serum urea concentration of 2.1 mmol/l

A diagnosis of SIAD (syndrome of inappropriate antidiuresis, also known as the syndrome of


inappropriate secretion of antidiuretic hormone) is often made without all the criteria being
satisfied. Clinically, there should be no evidence of fluid overload (oedema) or dehydration;
normal adrenal and renal function must be demonstrable; serum osmolality must be
decreased and, critically, the urine osmolality should be greater than that of the serum
(establishing that there is impaired water excretion). The cortisol concentration in this case,
given that the patient is sick, effectively excludes adrenal failure and satisfies one criterion.
Urea concentration is low: this is likely to be because of water overload.

Patients with SIAD are usually in sodium balance and sodium excretion is appropriate for
intake. Very low urine sodium would suggest that hyponatraemia was due to sodium depletion.

156.
Following a head injury, a 24-year-old patient develops polyuria and
polydipsia and is suspected of having cranial diabetes insipidus. He undergoes
a water deprivation test. Which one of the following responses would most
indicate a positive (abnormal) result?

A rise in plasma osmolality to 302 mmol/kg during water deprivation

Failure to concentrate the urine during water deprivation, but achievement


of urine osmolality of 720 mmol/kg following the administration of
desmopressin

Correct
answer

Failure to concentrate the urine either with water deprivation or following the Your
administration of desmopressin
answer

Failure to concentrate the urine with a plasma osmolality of 280 mmol/kg at


the end of the period of water deprivation

Weight loss of 5% during the investigation


Significant weight loss, an increase in plasma osmolality and a failure of urinary concentration
during water deprivation are all diagnostic of diabetes insipidus (DI) but do not differentiate
between this being cranial or nephrogenic in origin. In nephrogenic DI, however, there is
typically no response to desmopressin, whereas this causes the urine to become concentrated in
cranial DI. A low plasma osmolality at the end of the investigation, with persistently dilute urine,
suggests prior water loading or continued water intake. Note that cranial DI can be transient
following head injury.
157.
A 49-year-old woman presents complaining of fatigue and weight loss.
She was successfully treated medically for a prolactinoma 3 years ago, and it
is suspected that she might now have MEN-1. Which of the following
underlying conditions is she most likely to have?

Medullar carcinoma of thyroid

Parathyroid hyperplasia

Your answer

Pancreatic islet-cell tumour

Pituitary adenoma

Phaeochromocytoma
Parathyroid hyperplasia is seen in the vast majority of patients with multiple endocrine neoplasia
type 1 (MEN-1), and the hypercalcaemia that they cause is the most common presenting
feature of the condition. Hypercalcaemia may be discovered incidentally or because it is
symptomatic, but the symptoms are often non-specific. Islet-cell tumours (most frequently
gastrinomas) are present in about 40% of cases. Pituitary tumours occur in about 30% of
patients: over a half are prolactinomas. Medullary-cell carcinomas of the thyroid and
phaeochromocytomas occur in MEN-2 but not in MEN-1.
158.
A 51-year-old man presents with weight loss and is found on
examination to have hepatomegaly. He has facial telangiectasia. Urinary 5-

HIAA levels are found to be elevated. Given the diagnosis of carcinoid


syndrome, which additional clinical feature is the most likely to be present?

Abdominal pain

Diarrhoea

Correct answer

Hypertension

Skin rash

Your answer

Wheezing
Diarrhoea and flushing, occurring separately or together, are the most frequent presenting
features of carcinoid syndrome (approximately half of all patients). Abdominal pain occurs in
about 10%. Wheezing is a recognised feature of the condition but is uncommon, and a skin rash
(pellagra) due to niacin deficiency (niacin synthesis may be decreased in carcinoid syndrome) is
rare. Hypertension is not a feature of the condition.
159.
A 73-year-old woman presents with weight loss and is found to have a
serum calcium concentration of 3.22 mmol/l. A skeletal survey is normal. Nonmetastatic hypercalcaemia is suspected. Endoscopy reveals a gastric
carcinoma.
Secretion of which of the following substances by the tumour is most likely to be
responsible?

Calcitonin

Calcitriol (1,25-dihydroxycholecalciferol)

Osteoclast-activating cytokines

Your answer

Parathyroid hormone

Parathyroid hormone-related peptide

Correct answer

In patients with solid tumours, non-metastatic hypercalcaemia is most frequently the result of
secretion of parathyroid hormone-related peptide (PTHrP) by the tumour, and is reported in
association with gastric cancer. Non-parathyroid secretion of PTH itself is very rare. Unregulated
production of calcitriol has been described in patients with lymphoma but is rare. Secretion of
calcitonin, either ectopically or by medullary-cell carcinoma of the thyroid, is clinically silent: it
does not perturb calcium homeostasis. Secretion of osteoclast-activating cytokines is an

important cause of hypercalcaemia in patients with myeloma, but this is a relatively uncommon
malignancy.
160.
A 55-year-old man is found incidentally to have hypercalcaemia during
a routine health screen.
Which one of the following biochemical findings would be most suggestive of
this being caused by primary hyperparathyroidism rather than any other cause
of hypercalcaemia?

Elevated 24-h urinary calcium excretion

Your answer

Elevated serum alkaline phosphatase activity

Low serum concentration of calcitriol (1,25-dihydroxycholecalciferol)

Normal serum phosphate concentration

Serum PTH concentration within the normal range

Correct answer

Although parathyroid hormone (PTH) concentrations are often increased in patients with
hyperparathyroidism, they are not always so. PTH secretion should be suppressed by
hypercalcaemia from any other cause, so that a PTH value in the normal range is consistent with
the diagnosis. Urinary calcium excretion is increased in many causes of hypercalcaemia (except
familial hypocalciuric hypercalcaemia), including hyperparathyroidism. Serum alkaline
phosphatase activity can also be elevated with hypercalcaemia, regardless of the cause (with
the exception of myeloma). Serum phosphate concentrations tend to be reduced in
hyperparathyroidism (PTH is phosphaturic) and calcitriol concentrations to be increased (PTH
stimulates the formation of this hormone).
161.
A 57-year-old woman is brought to A&E after sustaining a head injury.
A skull X-ray shows there is no fracture, but that there is erosion of the
anterior clinoid processes of the pituitary fossa, suggesting a pituitary tumour.
Which of the following hormones is most likely to be being secreted in excess?

Adrenocorticotrophin (ACTH)

Follicle-stimulating hormone (FSH)

Growth hormone

Prolactin

Thyroid-stimulating hormone (TSH)

Your answer

Prolactinomas are the most frequently occurring functional pituitary tumours; however, prolactin
secretion may be increased in patients with other pituitary tumours because of the interruption
of the inhibitory effect of dopamine secreted by the hypothalamus. ACTH-secreting and growth
hormone-secreting tumours both occur less frequently and TSH- and FSH-secreting tumours are
uncommon. Non-functional tumours can occur at any age but are more frequent in the elderly.
162.
A 42-year-old patient complains of severe fatigue following surgery for
a pituitary tumour and is put on growth hormone replacement. Which of the
following is a well-recognised effect of this treatment?

Decrease in serum lipoprotein(a) concentration

Increase in fasting serum triglyceride concentration

Increase in fat mass

Increase in lean body mass

Your answer

Increase in serum total cholesterol concentration


Growth hormone replacement in adults with deficiency of the hormone has numerous effects,
including an increase in vitality and overall quality of life. Lean body mass tends to increase but
body fat decreases, often to a greater extent. Serum total- and LDL-cholesterol concentrations
and triglycerides tend to decrease but lipoprotein (a) concentration may increase.
163.
A 37-year-old man with a diagnosis of hypogonadotrophic
hypogonadism is being followed in the endocrine clinic. He does not desire
fertility at present. Which would be the most appropriate treatment at this
stage?

Oral testosterone replacement

Pulsatile subcutaneous administration of gonadotrophin-releasing hormone


(GnRH)

Regular injections of human chorionic gonadotrophin

Regular injections of human menopausal gonadotrophin (HMG)

Regular testosterone injections

Your
answer

If fertility is not required, there is no need to stimulate spermatogenesis with GnRH or


gonadotrophins: only testosterone replacement is required. Testosterone undecanoate is

available for oral use, but frequent dosage is required; and, because of poor absorption, the
plasma concentrations that can be achieved are often subnormal. Testosterone injections,
implants or patches are more reliable.
164.
A 44-year-old man is surprised to find that he cannot easily get his feet
into a pair of shoes that he last wore 5 years ago. He goes to buy a new pair
and is told that his size has increased. He trawls the Internet for an
explanation and, deciding that he may have acromegaly, consults his GP. The
GP has not seen him for several years and thinks his appearance has changed,
so refers him to the endocrine clinic.
Which of the following would be the most useful first-line test for investigating him?

Glucose tolerance test with measurement of growth hormone

Insulin hypoglycaemia test (insulin tolerance test, insulin stress test)

Measurement of serum growth hormone during sleep

Measurement of serum growth hormone following exercise

Measurement of serum insulin-like growth factor-1 (IGF-1) concentration

Your answer

In healthy individuals, growth hormone secretion is suppressed following the administration of


glucose. Failure of suppression is diagnostic of excessive growth hormone secretion. As an initial
screening test however, it may be more practical to measure serum IGF-1 and to follow this with
an OGTT if the results are abnormal. An advantage of IGF-1 is that levels are relatively stable
over time. The insulin hypoglycaemia test, measurement of growth hormone during sleep and
measurement following exercise, may be used in the investigation of suspected growth hormone
insufficiency.
165.
MR imaging indicates a microadenoma in a 36-year-old man presenting
with features of acromegaly and proven to have excessive growth hormone
secretion. Which of the following would usually be the treatment of choice?

Medical treatment with a growth hormone-receptor antagonist

Medical treatment with a somatostatin analogue

Radiotherapy

Transfrontal surgery

Your answer

Trans-sphenoidal surgery

Correct answer

Surgery is generally regarded as the first-line treatment for patients with acromegaly. The
trans-sphenoidal route is preferred except with large tumours, which may require a transfrontal
approach. Radiotherapy is sometimes used if surgery does not reduce growth hormone
concentrations to acceptable levels. Medical treatment is used principally as an adjunct to
surgery. Dopamine agonists (eg cabergoline) have been superseded by somatostatin agonists
(eg octreotide) and growth hormone-receptor antagonists (eg pegvisomant) are now becoming
available.
166.
A 49-year-old woman is investigated to determine the cause of
Cushings syndrome following the demonstration of hypercortisolaemia by a
high 24-hour urinary cortisol excretion. Her midnight plasma cortisol
concentration is elevated. Her 0900-h plasma cortisol concentration falls by
60% following a high-dose dexamethasone suppression test; plasma cortisol
concentration increases by 25% following intravenous corticotrophinreleasing hormone (CRH). Which is the most likely diagnosis?

Adrenal adenoma

Your answer

Adrenal carcinoma

Bilateral adrenal hyperplasia

Cushings disease (pituitary-dependent Cushings syndrome)

Correct answer

Ectopic secretion of ACTH


Incomplete suppression of cortisol following high-dose dexamethasone (2 mg 6-hourly for 48 h)
is typical of Cushings disease: there is typically no suppression in patients with other causes of
Cushings syndrome. An increase in cortisol following the administration of CRH is also
suggestive of Cushings disease, but with other causes there is usually no response.
Occasionally, carcinoid tumours secrete ACTH and cause similar responses in these tests to
those seen in Cushings disease. This, however, is a rare cause of Cushings syndrome.
167.
A 50-year-old woman with hypertension that has been difficult to
control with drugs is found to have hypercortisolaemia. Which one of the
following clinical findings would most suggest that ectopic secretion of ACTH
is the cause of the condition?

Glycosuria

Worsening hypertension

Hypokalaemia

Muscle wasting

Your answer

Weight loss

Correct answer

Glycosuria, hypertension, hypokalaemia and muscle wasting can occur with Cushings syndrome
from any cause. The most common manifestation is centripetal fat deposition, often with weight
gain, but weight loss suggests there is an underlying malignancy.
168.
An 18-year-old woman complains of a 2-month history of vague ill
health and nausea. She has had several episodes of dizziness and is found to
have postural hypotension. Which of the following investigations is required to
best demonstrate that these features are the result of adrenal failure?

Measurement of early morning and midnight plasma cortisol


concentrations

Measurement of early morning plasma ACTH (corticotrophin)


concentration

Measurement of 24-h urinary cortisol excretion

Your answer

Overnight dexamethasone suppression test

Short ACTH (Synacthen) stimulation test

Correct
answer

A diagnosis of adrenal failure is demonstrated by a failure of the plasma cortisol concentration to


increase in response to ACTH. Demonstration of a high plasma ACTH concentration indicates
primary rather than secondary (to pituitary insufficiency) adrenal failure. Random
measurements of cortisol are only of value in sick patients, in whom cortisol concentrations
should normally be high. Measurement of the diurnal variation in secretion (which is lost), the
cortisol response to dexamethasone (decreased) or urinary cortisol excretion (increased) are
used in the diagnosis of Cushings syndrome but have no place in the diagnosis of adrenal
failure.
169.
A 17-year-old young woman is admitted to A&E having collapsed at a
rave. She is in a shocked state and unable to give a coherent history, but is
found to have a card in her purse that indicates that she is on steroids for
adrenal failure. A clinical diagnosis of an addisonian crisis is made and a blood
sample is taken for cortisol measurement. Finger prick glucose testing reveals
a BM of 3.4 mmol/l. Which of the following should be given the most priority
in her management?

Intravenous glucose infusion

Parenteral administration of hydrocortisone

Your answer

Replacement of mineralocorticoid

Resuscitation with intravenous physiological saline and hydrocortisone

Correct answer

Treatment of any precipitating factor

All these measures (with the exception of mineralocorticoid replacement) are required in an
addisonian crisis, glucocorticoid and fluid replacements the priority, often given simultaneously
in clinical practice, but the immediate priority is fluid resuscitation. Mineralocorticoid
replacement may be required long term, but is unnecessary in the acute setting because the
large doses of hydrocortisone used supply mineralocorticoid activity.
170.
A 24-year-old man is found to have hypertension during an
examination for life assurance purposes. Over the next few months, this is
demonstrated to fluctuate considerably in severity and proves difficult to
control. Which of the following additional features would most suggest that a
phaeochromocytoma is causing his hypertension?

Diarrhoea

Flushing

Headache

Your answer

Muscle weakness

Tremor
Approximately 80% of patients with phaeochromocytoma complain of headaches, which are
often paroxysmal. Tremor is much less common. Pallor, not flushing, is seen in those with a
phaeochromocytoma and patients may complain of constipation. Flushing and diarrhoea are
features of the carcinoid syndrome. Approximately one-quarter of patients experience muscle
weakness, which can also be present in primary aldosteronism, another endocrine cause of
hypertension.
171.
A 39-year-old man with untreated hypertension has a plasma
potassium concentration of 2.8 mmol/l. When measured at 0900 h with the
patient supine, his plasma aldosterone concentration is elevated and renin

activity is low. When measured at 1200 h with the patient upright, the plasma
aldosterone concentration increases by 20%. Plasma cortisol concentration
was also measured at both times and found to be decreased by 50%.
What is the most likely diagnosis?

Adrenal adenoma secreting aldosterone

Bilateral adrenal hyperplasia

Correct answer

Liddles syndrome

Secondary aldosteronism

Steroid 11-hydroxylase deficiency

Your answer

Adrenal adenomas that secrete aldosterone (now thought to be the less likely cause of
hyporeninaemic hyperaldosteronism) are usually sensitive to ACTH, and aldosterone secretion
falls during the day as ACTH secretion (and that of cortisol) declines. With bilateral adrenal
hyperplasia, however, the aldosterone concentration is usually higher when the patient is erect
than when supine. Hypertension is also a feature of Liddles syndrome and steroid 11
-hydroxylase deficiency, but aldosterone concentrations are low. In secondary aldosteronism,
aldosterone secretion is increased secondary to an increase in renin secretion, and plasma renin
activity is normal or increased.
172.
A 52-year-old woman with primary hypothyroidism is being treated
with thyroxine replacement, the dose of which is being titrated against the
results of biochemical thyroid function tests. Two weeks after the last increase
in dose, the results of thyroid function tests are: free thyroxine 28 pmol/l
(normal 926); TSH 14 mU/l (normal 0.25.0). Which (if any) would be the
most appropriate next step in the management of this patient?

A decrease in the dose of thyroxine

A further increase in the dose of thyroxine

Your answer

No change in dose at this time

Correct answer

Question patients compliance with medication

Replace thyroxine with triiodothyronine

It is too soon after the last change in dose for a new steady state to have been achieved: it is
usual to wait for a month before reviewing the response. When thyroxine replacement is
started, the TSH concentration often falls to normal more slowly than free thyroxine increases.
Free thyroxine concentrations in clinically euthyroid patients on thyroxine replacement are
often high normal or even slightly elevated, reflecting the lack of production of
triiodothyronine (T3) by the thyroid: peripheral metabolism of thyroxine is the only source of
T3. Irregular medication could explain these results but should not be considered until
sufficient time has elapsed for the data reliably to reflect the effects of replacement.

Triiodothyronine is usually only used in very severe hypothyroidism, when a rapid response to
treatment is required.

173.
A 20-year-old woman presents with anxiety and weight loss with
increased appetite. Thyrotoxicosis is suspected and various investigations are
performed. Which of the following findings would most suggest that she has
Graves disease?

High ESR

High serum triiodothyronine (T3) concentration but normal thyroxine (T4)


concentration

High titre of thyroid peroxidase autoantibodies

Your
answer

Low thyroid uptake of technecium-99m

Normal serum TSH concentration


Antibodies to thyroid peroxidase and thyroglobulin are found in the serum of the majority of
patients with Graves disease, in which radioisotope uptake is typically increased. A low uptake
occurs in subacute (de Quervains) thyroiditis (in which the erythrocyte sedimentation rate
(ESR) is typically elevated). Elevated T 3 with a normal T4 (T3 toxicosis) can occur early in the
course of thyrotoxicosis from any cause. The high concentrations of thyroid hormones suppress
TSH secretion to very low levels in thyrotoxicosis due to thyroid disease: detectable (not always
elevated) TSH in thyrotoxicosis suggests that this is caused by a pituitary adenoma secreting
TSH.
174.
A 35-year-old woman had a febrile infection associated with a painful
swelling in her neck a week ago. Her thyroid function tests show evidence of
thyrotoxicosis and her ESR is raised. What is the most likely diagnosis?

Sporadic goitre

Hashimoto thyroiditis

Your answer

Fibromatosis

De Quervains thyroiditis

Correct answer

Follicular thyroid carcinoma

Subacute (or de Quervains) thyroiditis is due to thyroid infection by any of a number of


viruses, especially paramyxoviruses (mumps), coxsackieviruses, influenza viruses,
adenoviruses and echoviruses.

The most prominent symptom is pain in the thyroid, often radiating to the ears. A small,
tender goitre can be palpated that is usually diffuse, but there can be asymmetrical
involvement. Systemic upset with fever is variable but sometimes profound, and symptoms of
a prodromal viral infection several weeks earlier may be recalled. There is a granulomatous
thyroid inflammation with follicular destruction, and the release of thyroid hormones often
results in a transient thyrotoxicosis, lasting for 14 weeks. Continuing thyroid destruction then
leads to a phase of hypothyroidism once stored hormone is depleted. This lasts 412 weeks
before euthyroidism is restored, but relapses occur in 1020% of cases. Sometimes only one
phase of thyroid disturbance is seen.

Confirmation of the clinical diagnosis is made by finding an elevated erythrocyte sedimentation


rate (ESR) and low or absent radioiodine uptake by the thyroid. Thyroid function requires
continuous monitoring as the disease evolves. Mild cases may resolve spontaneously with
aspirin as symptomatic treatment, but most patients benefit from prednisolone 4060 mg
daily, which rapidly alleviates the pain. The dose is tapered over 68 weeks, depending largely
on symptoms. Propranolol may be useful for thyrotoxic symptoms, and temporary thyroxine
replacement is sometimes needed during the hypothyroid phase.

175.
A 60-year-old woman presented complaining of a 6.3 kg (1 stone)
weight loss, polyuria and depressive mood for 8 weeks. The following
laboratory results are obtained: calcium 3.4 mmol/l, phosphate 1.1 mmol/l,
parathyroid hormone 5 ng/l (reference < 60 ng/l). What is the most likely
cause for her complaints?

Primary hyperparathyroidism

Your answer

Secondary hyperparathyroidism

Hypercalcaemia due to cancer

Correct answer

Vitamin D deficiency

Hyperthyroidism

The causes of hypercalcaemia can be classified according to whether serum parathyroid


hormone concentrations are elevated (i.e. primary hyperparathyroidism) or low (i.e. not due
to a parathyroid tumour). Primary hyperparathyroidism and malignancy are the most common
causes and account for more than 90% of patients with hypercalcaemia.

A detailed clinical history and examination will usually help to differentiate between these two
diagnoses. In primary hyperparathyroidism, the hypercalcaemia is often less than 3 mmol/l,
asymptomatic and may have been present for months or years. However, in malignancy, the
patients are usually acutely ill, often with neurological symptoms, the hypercalcaemia is more
than 3 mmol/l and the cancer (eg lung, breast or myeloma) is often readily apparent.

176.
A 17-year-old young woman has been referred by her gynaecologist.
She has been complaining of amenorrhoea for 5 months, although no
gynaecological abnormality has been found. She feels well and is very active
but her weight has decreased from 61 kg to 43 kg in the last 6 months. Her
height is 168 cm. On examination her BP is 90/60 mmHg, heart rate 64 bpm.
What is the most likely diagnosis?

Conns syndrome

Crohns disease

Anorexia nervosa

Your answer

Hyperthyroidism

Diabetes mellitus

Patients with anorexia nervosa actively maintain an unduly low body weight. For diagnostic
purposes, an unduly low body weight may be defined as a weight at least 15% below that
expected for the persons age, height and sex, or as a body mass index below 17.5.
Amenorrhoea (in postmenarchal women who are not taking an oral contraceptive) is almost
always present in these patients.
177.
A patient has been referred by her GP because she has been
complaining of frequent episodes of sweating and palpitations associated with
a low blood glucose level. The family history reveals a brother with type-1
diabetes. A blood test shows the following results: glucose 1.1 mmol/l, insulin

> 500 pmol/l (reference 15100 pmol/l) and C-peptide of < 0.2 nmol/l
(reference 0.21.4 nmol/l). What is the most likely diagnosis?

Type-1 diabetes mellitus

Insulinoma

Pancreatic carcinoma

Factitious insulin-induced hypoglycaemia

Your answer

Maturity-onset diabetes of the young (MODY)


Factitious insulin-induced hypoglycaemia is as common in previously healthy subjects as in
insulin-dependent diabetics and is due to the deliberate, but concealed, injection of insulin. The
history suggests insulinoma, but this is eliminated by the laboratory results that reveal high
plasma insulin and low C-peptide (and proinsulin) concentrations during hypoglycaemia. In
longstanding factitious hypoglycaemia, and in insulin-treated diabetics, insulin antibodies may
be present in the plasma. Although once considered a strong pointer to factitious
hypoglycaemia, the presence of insulin antibodies should nowadays suggest autoimmune insulin
syndrome.
178.
A 55-year-old obese patient with hypercholesterolaemia (LDL 5.2
mmol/l, HDL 1 mmol/l), well-controlled type-2 diabetes and hypertension has
been on a low cholesterol diet for the last 6 months. His latest LDL level is 4.8
mmol/l, and triglycerides are within the normal range. In terms of primary
prevention, what is the next therapeutic step in his management?

Add pioglitazone

Add gemfibrozil

Add clopidogrel

Add statins

Your answer

Add ezetimibe

The latest European guidelines suggest that LDL cholesterol should be < 4.5 mmol/l rather than
< 5 mmol/l in secondary prevention, while evidence from primary prevention suggests that
similar targets should also apply to primary prevention. Statins are the medication of choice. As

targets are reduced and further evidence from the lowdosehigh-dose statin comparator trials
are announced, it is likely that targets will fall further and that combination therapy for
hyperlipidaemia will become routine as is the case in hypertension. The challenge will still
remain with implementation, but the benefits of satisfactory risk-factor control are that both
morbidity and mortality from cardiovascular disease will be decreased.
179.
50-year-old woman has been diagnosed with a pericardial effusion.
Which endocrine disease is most likely to be associated with this finding?

Hyperthyroidism

Hypothyroidism

Your answer

Phaeochromocytoma

Chronic renal insufficiency

Hypogonadism

Clinically silent pericardial effusion is common in untreated hypothyroidism, it may be seen in


hyperthyroidism but incidence is less. The effusion itself has a high cholesterol content, which
may produce an unusual secondary pericarditis with cholesterol deposits of gold paint
appearance. The pericardial effusion very rarely needs to be treated in its own right, and
subsides when thyroid replacement therapy is given.
180.
What is the most reliable investigation for determining the volume of
the thyroid gland?

Thyroid isotope uptake technetium-99m

Ultrasound

Correct answer

X-ray of the neck

Thyroid isotope uptake iodine-125

PET scan

Your answer

Ultrasound is useful in determining thyroid size accurately. Thyroidal uptake of radioisotopes


(especially technetium-99m) is indicated if destructive thyroiditis is suspected as a cause of
goitre. Otherwise, the major role for imaging is to ensure there is no tracheal compression or

intrathoracic/retrosternal component in a patient with suggestive symptoms, and a CT scan is


then the preferred investigation.
181.
A 30-year-old man presents with a 3-month history of deteriorating
physical performance at work, associated with dysarthria and clumsiness. On
examination he looks anaemic, has hepatomegaly and KayserFleischer rings
in the cornea. What would be the most important investigation to support the
suspected clinical diagnosis?

CT scan of the brain

MRI scan of the brain

Liver biopsy

Serum ceruloplasmin level

Your answer

CRP level

Serum ceruloplasmin can be measured enzymatically (copper oxidase), by radial


immunodiffusion or by reverse passive haemagglutination. The activity and concentration of this
glycoprotein is reduced or absent (less than 200 mg/l) in 95% of patients with Wilsons disease.
In the absence of Kayser-Fleischer rings or neurologic abnormalities, a liver biopsy for
quantitative copper determination would be essential to establish the diagnosis of Wilson's
disease, but this would likely not be necessary here given the typical clinical picture.
182.
A 30-year-old man presents with a 3-month history of deteriorating
physical performance at work, associated with dysarthria and clumsiness. On
examination he looks anaemic, has hepatomegaly and Kayser-Fleischer rings
in the cornea. What is the most likely diagnosis?

Hepatitis C infection

Wilsons disease

Alcohol abuse

Motor neurone disease

Subacute sclerosing panencephalitis

Your answer

Wilsons disease may present in childhood, adolescence or early adulthood. Symptoms and signs
may be clinically undetectable under 5 years of age, and few present after the age of 35 years,
although diagnosis over 55 years has been reported. In 90% of patients, the disease presents
with juvenile hepatic disease or with neurological/psychiatric manifestations. In large studies of
patients with Wilsons disease, initial manifestations were: hepatic (40%); neurological (30%);
psychiatric (10%); haematological (12%); and renal (1%). Some 25% of patients have two or
more organs involved (usually liver and brain) at the initial assessment. KayserFleischer rings
are almost pathognomonic of Wilsons disease.
183.
A 30-year-old man presents with a 3-month history of deteriorating
physical performance at work, associated with dysarthria and clumsiness. On
examination he looks anaemic, has hepatomegaly and KayserFleischer rings
in the cornea. Given the likely diagnosis, what would be the most important
treatment option?

Blood transfusion

d-Penicillamine

Your answer

Ciclosporin

Azathioprine

Interferon-alpha

D-Penicillamine is effective in removing copper from patients with Wilsons disease. The
optimum time for treatment is in the early stages, and all patients with Wilsons disease
should be treated, even if asymptomatic. Treatment is lifelong, unless the patient undergoes
liver transplantation. The aim of treating a patient with Wilsons disease is to reduce toxic
copper levels in the body tissues. This can be achieved by increasing the urinary excretion of
copper. A negative copper balance should be monitored carefully since, with an increase in
urine and faecal copper excretion that exceeds copper intake, increased urinary copper may
reflect increased plasma non-ceruloplasmin copper (copper in its most damaging form).

The effect of any therapy should be regularly monitored by clinical and radiological
assessment, and by biochemical monitoring of abnormal liver enzymes and liver function.

184.
A patient was referred by his GP because of a borderline fasting
glucose. You arrange for a glucose tolerance test to clarify the diagnosis. For
this test how much glucose is dissolved in 250 ml of water?

10 g

50 g

75 g

Your answer

100 g

150 g

After an overnight fast, the subject drinks 75 g of anhydrous glucose dissolved in 250 ml water,
venous blood is sampled at baseline and 2 hours later. Food intake should be normal during the
preceding few days: poor nutrition can cause delayed hyperglycaemia with a raised 2-hour value
(the lag curve).
185.
A patient with longstanding type-2 diabetes was found to have a
urinary albumin excretion rate of 400 mg/l. His diabetes is well controlled and
he is normotensive. What additional drug should he be prescribed?

Aspirin

Atenolol

Lisinopril

Your answer

Clopidogrel

Pioglitazone

In patients with type-1 diabetes with a mean entry blood pressure of 122/77 mmHg, a
combined analysis of one European and one American study (total n = 225) showed an adjusted
risk reduction of 63% (95% CI, 1684%; p = 0.017) for the development of clinical
nephropathy comparing the ACE inhibitor captopril 100 mg/day with placebo. Three smaller
studies in normotensive type-2 patients have reported a similar reduction in the rate of
development of clinical nephropathy. Thus blockade of the reninangiotensin system by any
means appears to confer benefit.
186.
A patient presents with truncal obesity, insulin resistance and
dyslipidaemia. What additional clinical feature might you expect to be
present?

Asthma

Renal failure

Hypertension

Your answer

Cancer

Ophthalmoplegia

Metabolic syndrome X is the term given to the co-occurrence of insulin resistance and glucose
intolerance (ranging from mild to overt type-2 diabetes), with truncal obesity, dyslipidaemia
(raised triglycerides and a high LDL: HDL ratio) and hypertension.
187.
A patient has been complaining of a 2-month history of intermittent
flush associated with tachycardia and wheezing. There have also been
episodes of profuse watery diarrhoea. Given the suspected diagnosis, what
would be the most appropriate investigation?

Dexamethasone suppression test

Urinary catecholamine collection

Urinary 5-hydroxyindoleacetic acid collection

Your answer

Abdominal ultrasound

24-hour ambulatory blood pressure recording

The biologically active metabolite characteristically produced by metastatic carcinoid tumours is


5-hydroxytryptamine (5-HT, serotonin), synthesised from the amino acid tryptophan. 5-HT plays
a part in the pathogenesis of some of the symptoms of the carcinoid syndrome, particularly the
diarrhoea and bronchoconstriction. It is metabolised to 5-hydroxyindoleacetic acid (5-HIAA),
which accounts for 95% of the urinary excretion of 5-HT.
188.
A 55-year-old woman has been complaining of a 6-month history of
weight gain. She is otherwise well and takes no medication. On examination
her BMI is 28, BP 170/100 mmHg, she has a round red face and a slight
atrophy of her arm muscles. Renal function test and urinalysis are normal.
What is the next step in obtaining the diagnosis?

Dexamethasone suppression test

Urinary catecholamine collection

Your answer

Urinary 5-hydroxyindoleacetic acid collection

Abdominal ultrasound

CT scan abdomen

Low-dose dexamethasone suppression tests the diurnal circadian rhythm.


In normal subjects, the cortisol concentration begins to rise at 0200 h, peaking at 05300930 h.
Concentrations drop during the morning and reach a nadir at around 2400 h (cortisol < 50
nmol/l). Blood is taken for ACTH and cortisol at 0900 h and 2400 h. The finding of a normal
circadian rhythm virtually rules out the presence of active Cushing's syndrome.
189.
A patient with poorly diet-controlled type-2 diabetes mellitus needs to
be started on medication. Which concomitant condition would be a
contraindication for starting metformin?

Hypertension

Respiratory insufficiency

Your answer

Hyperlipidaemia

Hyperthyroidism

Adipositas

Lactic acidosis is best known in diabetic patients as a rare, but often fatal, complication of the
biguanide metformin, which acts mainly by inhibiting hepatic gluconeogenesis. The risk is much
increased in patients with respiratory insufficiency.

You might also like